Содержание

Задачи по математике 6 класс с решением и ответами

Предлагаем Вашему вниманию несколько математических задач для 6 класса с решением и ответами. Улучшите свои знания!

Задачи по математике для 6 класса и решения к ним

Как расставить 7 стульев у 4 стен комнаты, чтобы у каждой стены их было поровну?

Решение: Представьте себе комнату. Да, 1 стул может касаться одновременно двух стен. Если стулья расставить по углам, то будет использовано на 3 стула меньше. 4 (углы) — 3 (нужно еще стульев) = 1 (стул будет стоять на углу). Вот так выглядит решение:

Круг — это стул

Четверо мальчиков — Алеша, Гриша, Боря и Ваня — соревновались в беге. Три мальчика сказали правду, а 1 соврал. Алеша: — Я не был ни первым, ни последним. Гриша: — Я последний. Боря: — Я не был последним. Ваня: — Я был первым. Кто соврал и кто выиграл?

Предположим, что соврал Алеша. Значит, он был первым или последним. Тогда Гриша и Ваня скажут правду, но это будет не так.

Боря тоже говорит правду, так как Гриша говорит, что он последний. Гриша тоже по-любому говорит правду. Поэтому обманывает Ваня.

Для закрепления знаний решите задачу ниже.

Один из попугаев всегда говорит правду, другой всегда врет, а третий — середняк (иногда говорит правду, а иногда врет. Я спросил: Кто Кеша? Ответы: Гоша: он лжец. Кеша: Я хитрец! Рома: Он честный. Кто хитрец и кто лжец?


Придумайте трехзначное число, у которого с любым из чисел 142, 543 и 562 совпадал ровно 1 разряд.

Это просто: выпишем количество единиц, десятков и сотен в числах. Занимать данную позицию в вашем числе будут те, которые не повторяются.

  • Единицы — 2 3 2
  • Десятки — 4 4 6
  • Сотни — 1 5 5

И наше число будет 163.


3 мальчика нашли 27 ягод. Витя, сколько ягод нашел ты? — спросил Вася. — Вдвое больше, чем ты и Гена вместе. А Гена нашел в 2 раза больше ягод, чем ты. Сколько ягод нашел Вася?

Пусть ягоды Васи будут x, тогда ягоды Гены будут 2x, а ягоды Вити — 6x, то:

x + 2x + 6x = 27 9x = 27 x = 3

Таким простым уравнением понятно, что Вася нашел 3 ягоды.

Занимательные задачи

1 260

Занимательные задачи!

Чем хороши занимательные задачи  — ими можно интересно занять детей по в дороге, по пути в школу или устроить конкурс на школьном празднике. Обратите внимание, что мало кто сможет дать правильный ответ сразу, потому  не забывайте о маленьких подсказках, разгадывание задачек от этого будет не менее интересным.

Занимательные задачи по математике

1.В каждом из 4 углов комнаты сидит кошка. Напротив каждой из этих кошек сидят три кошки. Сколько всего в этой комнате кошек?

2. У отца шесть сыновей. Каждый сын имеет сестру. Сколько всего детей у этого отца?

3. В мастерской по пошиву одежды от куска сукна в 200 м ежедневно, начиная с 1 марта, отрезали по 20 м. Когда был отрезан последный кусок?

4. В клетке находятся 3 кролика. Три девочки попросили дать им по одному кролику. Каждой девочке дали кролика. И все же в клетке остался один кролик. Как так получилось?

5. 6 рыбаков съели 6 судаков за 6 дней. За сколько дней 10 рыбаков съедят 10 судаков?

6. На одном дереве сидело 40 сорок. Проходил охотник, выстрелил и убил 6 сорок. Сколько сорок осталось на дереве?

7.Два землекопа за 2 часа работы выкопают 2 м канавы. Сколько нужно землекопов, чтобы они за 100 часов работы выкопали 100 м такой же канавы?

8. Два отца и два сына разделили между собой 3 апельсина так, что каждому досталось по одному апельсину. Как это могло получиться?

9. По стеблю растения, высота которого 1 м, от земли ползет гусеница. Днем она поднимается на 3 дм, а ночью опускается на 2 дм. Через сколько суток гусеница доползет до верхушки растения?

10. Есть два ведра емкостью 4 и 9 литров. Как с их помощью принести из речки ровно 6 литров воды?

Ответы:

1.4

2.Одной девочке дали клетку с кроликом.

3.9марта

4.7

5. 6 рыбаков за день едят 1 судака.
Один рыбак есть 1/6 судака в день.
10 рыбаков едят за день 10/6 судака.
10 судаков делим на 10/6 судака = 6 дней

6.Все улетели

7. 2

8. Дед, отец и внук = 2 отца и 2 сына

9.Через 7/12 суток.

10. Из полного девятилитрового ведра нужно вылить в реку 8литров воды, пользуясь ведром в 4 литра. Затем литр, оставшийся в большом ведре, нужно перелить в пустое четырехлитровое ведро. Если в него теперь добавить три литра из полного большого ведра, то в девятилитровом ведре как раз останется шесть литров воды.

Подумай и сосчитай

Чтоб одеть тепло сыночков,
Не хватает двух носочков.
Сколько же в семье сынков,
Если в доме шесть носков?
Ответ:четверо

Столько книжек у ребяток,
Сколько у Алеши пяток.
Принесла ребяткам Галя
Мячик, книжку, мишек.
Вы, ребята, посчитали,
Сколько стало книжек?
Ответ: три
******************
К трем лягушкам у болота
Прибежали два енота,
Прискакала тетя жаба
И пришла наседка Ряба.
Сколько в камышах болотных
Оказалось земноводных?
Ответ: четверо

Задачи на внимание

1. Подумай и скажи — кто быстрее переплывет речку — утята или цыплята?

2. Подумай и скажи — какого цвета волосы у колобка?

3. Отгадай загадку:
Лежали конфетки в кучке.
Две матери, две дочки
Да бабушка с внучкой
Взяли конфет по штучке,
И не стало этой кучки.
Сколько конфет было в кучке?

4. Росли 5 берез. На каждой березе по 5 больших веток. На каждой ветке по 5 маленьких веток. На каждой маленькой ветке — по 5 яблок. Сколько всего яблок?

5. Подумай и скажи — что помогает выжить белым медведям в пустыне, где нет воды?

6. На каких деревьях вьют свои гнезда страусы?

7. На столе лежит 2 яблока и 4 груши. Сколько всего овощей лежит на столе?

8. Подумай и скажи — кто громче рычит: тигр или буйвол?

9. Посмотрел Ваня утром в окно и говорит:

— А на улице, оказывается, очень сильный ветер. Нужно теплее одеваться.
Как он догадался, что на улице ветер? Что он увидел?

10. Пошли 2 девочки в лес за грибами, а навстречу 2 мальчика. Сколько всего детей идет в лес? (подсказка: 2 — остальные идут обратно)

11. В комнате горело 5 свечей. Зашел человек, потушил 2 свечи. Сколько осталось? ( подсказка: 2- остальные сгорели)

12. Бревно распилили на 4 части. Сколько сделали распилов?

13. Прочитай слова и скажи — какое слово лишнее в каждом ряду?
— диван, стул, шкаф, конура, тумбочка,
— гвоздика, ромашка, камыш, лилия, астра,
— боровик, мухомор, сыроежка, подберезовик, лисичка.

14. Подумай и скажи — сколько земли будет в яме глубиной 1 метр, длиной 1 метр и шириной 1 метр?

15. У шестилетней девочки была кошка с коротким хвостом. Она съела мышку с длинным хвостом, а мышка проглотила 2 зернышка и съела тонкий кусочек сыра. Скажи, сколько лет было девочке, у которой была кошка?

16. На одном берегу реки стоит петух, а на другом индюк. Посреди реки — островок. Кто из этих птиц быстрее долетит до островка?

17. Скажи сколько грибов можно вырастить из 5 семечек?

18. Скажи, кто обитает в море на большей глубине: щука, рак или форель?

19. Гусь на двух ногах весит 2 кг. Сколько он будет весить, стоя на одной ноге?

20. На клене 5 веток. На каждой ветке по 2 яблока. Cколько яблок на клене?

Мой блог находят по следующим фразам

решение задач за 5 — 6 класс — Колпаков Александр Николаевич

На этой странице публикуются решения задач по математике для 5 и 6 класса: части, проценты, пропорции, вычисления, простые текстовые задачи на движение, на работу, не требующие применения никаких уравнений кроме линейных. Помните о том, что виртуальный репетитор по математике не знает

по какой программе учится Ваш ребенок и поэтому возможны расхождения со школой. Часто одну и ту же задачу на дроби можно решить по разному: средствами 5 класса (при помощи отдельных действий с числителями и знаменателями), а можно, например, средствами 6 класса, выполняя умножение или деление на соответствующие дроби. Для того, чтобы помочь репетитору математики выбрать оптимальный способ оформления номера, указывайте ссылки на авторов школьных учебников и Ваш класс. Пожалуйста, не заваливайте репетитора целыми списками номеров. Ориентировочное ограничение: 1-2 номера для каждого посетителя. Если Вам понравилась эта страница — нажмите на кнопку +1:
Это поможет другим ученикам найти сайт в интернете.

Виртуальный репетитор по математике (5-6 класс). Решения ваших задач.

Вопрос от Вовы: Из пункта М в пункт N выехал почтальон со скоростью 23 км/ч, и одновременно с ним из N в M выехал второй почтальон со скоростью 19 км/ч. Когда первый почтальон прибыл N, второму еще оставалось до М проехать 24 км. Каково расстояние между М и N?

Репетитор по математике о задаче про почтальона (А.Н. Колпаков)
Обозначим буквой t время, за которое первый почтальон прибыл в N, тогда 23t — путь, пройденный первым, а 19t — путь, пройденный вторым почтальоном за это же время. Так как второму езе оставалось 24 км, то он прошел за это время расстояние на 24 км меньшее, чем первый, поэтому 23t-19t=24. Решим это простенькое уравнение и получим в ответе t=6 часов. В итоге (км) — пусть первого, равный всему расстоянию от M до N.
Ответ: 138 км.

Вопрос репетитору по математике от Оксаны: Помогите с задачей. Она элементарная, но нам надо ее решить без использования дробей!!! У квадрата одну его сторону увеличили на 9 см, а другую сторону уменьшили в 5 раз. В результате этого получилcя прямоугольник с периметром равным 66 см. У какой фигуры — у прямоугольника или у квадрата — получилась больше площадь и на сколько?

Репетитор по математике о задаче c квадратом:
Если Вы хотите решить эту задачу без применения каких-либо дробей, не выходя за рамка программы 5 класса, то буквой икс необходимо обозначить наименьшую из величин, то есть ширину прямоугольника. Итак, пусть AK=x, тогда AD=AB=5x. Поскольку сторону AB увеличили на 9 см, то длина полученного прямоугольника выражается как 5x+9. Принимая во внимание условие с периметром, получаем простенькое уравнение без дробей:
x+x+5x+9+5x+9=66
Решая его получим, что x=4. Теперь легко найти интересующие нас площади: кв.см., кв.см.
И тогда 400-116=284 кв.см. — разница между ними.

Вопрос от Анны: Помогите решить задачу.
Отец и сын, работая вместе, покрасили забор за 12 ч. Если бы отец красил забор один, он выполнил бы эту работу за 21 ч. За сколько часов покрасил бы этот забор сын?

Репетитор по математике, Тимур Розугнов
Примем весь объем работы (забор) за единицу и воспользуемся тем, что совместная скорость равна сумме отдельных скоростей отца и сына. Следить за решением удобно при помощи табличного метода оформления:
1) (заб/час) — совместная скорость
2) (заб/час) — скорость отца
3) (заб/час) — скорость сына
4) (часов) — время работы сына
Ответ: 28 часов

Вопрос от Марины:
Редактор прочитал две пятых рукописи, что составило 80 страниц. На другой день он прочитал четверть оставшихся страниц. ВОПРОСЫ: 1) Сколько страниц в рукописи? 2) Сколько страниц осталось не прочитано?

Репетитор по математике, Никита Афанасьевич
Для лучшего усвоения решения полезно сделать краткую запись. Выглядеть она будет следующим образом:

Решение:
1) (страниц) в рукописи.
2) (рукописи) — составляет остаток.
3) (страниц) — остаток.
4) (страниц) — прочитано во второй день.
5) (страниц) не прочитано.
Ответ: 90 страниц.

Задача от Наташи:
Мотоциклист в первый час проехал 3/8 всего пути ,во второй час 3/5 остатка,а в третий час остальные 40 км. Найдите весь путь. Помогите решить!

Репетитор по математике, Александр Колпаков
Старайтесь указывать для какого класса и по какой программе репетитору оформлять решение !!! будем считать, что что вы в 6 классе. Оформим краткую запись ровно так, как я это рекомендую делать своим ученикам: (в вертикальную рамку я выделяю доли, связанные законом сложения)

1) (остатка) — проехал мотоциклист за третий час
2) (км) — остаток
3) (всего пути) — остаток после пройденного мотоциклистом пусти за I час.
4) (км) — составляет весь путь
Ответ: 160 км

Вопрос от Оксаны: Объясните, пожалуйста, как правильно решить задачу: поезд проходит расстояние АВ за 10,5 ч. На сколько процентов следует увеличить его скорость, чтобы то же расстояние он преодолел за 8 ч? Решение нужно СРОЧНО к 1 сентября! Пыталась сама решить ее через уравнение, но не знаю правильно ли.

Репетитор по математике, Григорий Александров: Не нужно никаких уравнений. Они только Вас запутают. Вот мое решение: поскольку прирост любой вличины в процентах не зависит от ее единицы измерения, то примем за единицу полное расстояние от А до В. Тогда скорости будут такими: и Тогда прирост по скорости составит
Найдем какую часть эта величина составляет от прежней скорости:
Осталось эту часть перевести в проценты умножением на 100. Получим в итоге %

Задача от Арины:
У Шынар в копилке 80 монет достоинством 20 и 50 тенге Всего 2590 тенге. Сколько монет в копилке у Шынар достоинством 20 тенге? достоинством 50 тенге? Заранее спасибо очень надеюсь на вашу помощь.

Репетитор по математике, Колпаков А.Н.
Если бы все монеты были по 50 тенге, то Шынар имела бы всего 4000 тенге. Замена одной монеты в 50 тенге на одну монету достоинство в 20 тенге приводит к снижению капитала ровно на 30 тенге. На сколько тенге нам необходимо уменьшить общий капитал Шынар с 4000 до 2590? Ровно на 4000—2590=1410 тенге. Тогда сколько раз необходимо произвести замену? 1410:30=47 раз. Поэтому 47 монет нужно поменять на двадцатитенговые. Останется 80-47=33 монеты по 50 тенге.
Ответ: 47 монет по 20 тенге и 33 монеты по 50 тенге.

Вопрос от Татьяны: нужно решить задачу:
В первый день садовод вскопал на 40% своего участка, а во второй — 40% оставшейся части. На третий день он закончил работу, вскопав 180 кв.м. Определить площадь всего участка?

Репетитор по математике и физике, Галкин Р.А.
Можно предложить 3 способа решения. Остановлюсь на том, который ориентирован на 5 класс. В целях лучшего восприятия задачи составим схему (краткую запись) условия:Здесь все проценты переведены в дроби . Найдем какую часть (или сколько процентов) составляет вскопанная часть в 3 день от того, что осталось вскопать после 1-го дня:
1) %(остатка) -вскопали в 3 день.
По известному значению 180кв.м дроби найдем целую величину, то есть остаток:
2) (кв.м) — осталось после 1 дня
Найдем какую часть остаток составляет от всего участка:
3) %(всего участка) — осталось
По известному значению 300 кв.м дроби найдем целую величину, то есть весь участок:
4) (кв.м) — площадь всего участка.
Ответ: 500 кв.м.

Вопрос от Ангелины:
У меня возник вопрос с решением задачи. Помогите пожалуйста. Можно ли из какого угодно кол-ва троек получить в ответе 100, при помощи действий сложение, вычитание и умножение?

Репетитор по математике, Файгойз М.Ю.
Не очень понял вопрос. Что значит из «какого-угодно»? Угодно нам или угодно составителю задачи? Эх … не математик условие писал. Если на нас спускается количество троек как приказ, то не из любого. Ведь из двух троек никак нельзя составить 100. А если мы сами вправе выбирать количество троек, то можно так: . Конечно, условие должно быть переписано: можно ли из какого-нибудь количества троек получить 100?

Pages: 1 2 3

Разные задачи. Математика, 6 класс: уроки, тесты, задания.

1. Сметанный крем

Сложность: лёгкое

2
2. Пешеход и велосипедист

Сложность: среднее

2
3. Призовой фонд

Сложность: среднее

3
4. Сумма на счёте через год

Сложность: среднее

2
5. Доход с суммы

Сложность: среднее

2
6. Проценты по вкладу

Сложность: среднее

3
7. Расстояние до базы отдыха

Сложность: среднее

6
8. Ярмарка

Сложность: сложное

7
9. Насосы и вода

Сложность: сложное

4
10. Задача на составление уравнения

Сложность: среднее

4

Олимпиадные задания по математике 6 класс

                Вар-т 1            Вар-т 2            Вар-т 3

Задание 1.

Используя шесть раз цифру 2, знаки действий и скобки, напишите выражение,
значение которого равно 100.

Ответ: возможное решение (222 — 22) : 2 = 100

Задание 2.

На полке в один ряд стоят книги. Энциклопедия стоит пятой слева и семнадцатой справа.
Сколько книг на полке?

Ответ: 21 книга. (4 + 1 + 16 = 21)

Задание 3.

По углам и сторонам квадрата вбиты колышки на расстоянии 2 метра друг от друга.
Сколько колышков вбито, если сторона квадрата равна 10 метрам?
Показать решение на рисунке.

Ответ: 20 колышков.

Задание 4.

В забеге участвовало 37 человек.
Число спортсменов, прибежавших раньше Игоря, в 5 раз меньше числа тех, кто прибежал позже.
Какое место занял Игорь?

Ответ: 7 место. (х + 5х + 1 = 37; 6х = 36; х = 6.

Задание 5.

В коробке 14 белых и 14 чёрных шариков.
Какое минимальное количество шариков нужно достать из коробки,
чтобы среди них наверняка оказалось 2 черных шарика?

Ответ: 16. (14 + 2 = 16).

Задание 6.

В семье четверо детей, им 5,8,13и 15 лет. Детей зовут Аня, Боря, Вера, Галя.
Сколько лет каждому ребенку, если одна девочка ходит в детский сад, Аня старше Бори
и сумма лет Ани и Веры делится на 3?

Ответ: Вере — 5 лет; Боре — 8 лет, Ане — 13 лет; Гале — 15 лет.

Задание 7.

Младший брат Насти во время игры вырвал из книги 3 листа.
Настя сложила номера всех вырванных 6 страниц и получила 2016.
Докажите, что при сложении девочка допустила ошибку.

Ответ: сумма номеров страниц на одном листе число нечетное,
тогда сумма номеров 3-х листов тоже нечетное число.

Олимпиадные задания по математике 6 класс с решением


Задание 1.

В записи (88888888) нужно поставить знаки сложения таким образом,
чтобы получилась сумма, которая будет равна 1000.

Решение:
Способ 1: 88 + 8 + 8 + 8 + 888 = 1000
Способ 2: 8 + 8 + 888 + 88 + 8 = 1000.

Задание 2.

Малыш, Алиса, Кай и Женя заняли первые четыре места в соревнованиях,
причем никто из них не делил между собой какие-нибудь места. Известно:
Малыш не был ни первым, ни четвертым.
Алиса заняла второе место.
Кай не был последним.
Какое место занял каждый?

Ответ: Малыш — 3, Алиса — 2, Кай — 1, Женя — 4 место.

Задание 3.

Мама дала Зое денег, чтобы она в школьном буфете купила завтрак.
Когда Зоя вер вернулась из школы, то перед мамой отчиталась так:
1/2 всех денег я истратила на бумагу, 1/5 — на чай, а 3/10 — на конфеты.
Мама догадалась, что дочь истратила все деньги. Как она узнала?

Решение: 1/2 + 1/5 + 3/10 = 1, т.е. все деньги.

Задание 4.

Змей Горыныч побежден! — такая молва дошла до Микулы Селяниновича.
Он знал, что мог это сделать либо Илья Муромец, либо Алеша Попович, либо Добрыня Никитич.
Вскоре Микуле сообщили:
Змея Горыныча победил не Илья Муромец;
Змея Горыныча победил Алеша Попович.
Спустя некоторое время выяснилось, что одно их этих сообщений неверное, а другое верное.
Догадайтесь, кто из трех богатырей победил Змея Горыныча.

Ответ. Добрыня Никитич.
Решение.
Предположим, что Змея Горыныча победил Илья Муромец.
Тогда оба сообщения неверные-результат не соответствует условию задачи.
Предположим, что Змея Горыныча победил Алеша Попович. Тогда оба сообщения верные.
И этот результат не соответствует условию задачи.
Предположим, что Змея Горыныча победил Добрыня Никитич.
Тогда первое сообщение верное, а второе — неверное. Результат соответствует условию задачи.

Задание 5.

Трое рыбаков поймали 75 карасей. Стали варить уху.
Когда один дал 8 карасей, а другой 12, а третий 7, то карасей у них стало поровну.
Сколько карасей поймал каждый рыбак?

Решение.
75 — 8 — 12 — 7 = 48(осталось всего окуней).
48 окуней на 3 рыбака. 48 : 3 = 16.
У каждого рыбака осталось по 16 окуней.
16+ 8 = 24 — поймал 1 рыбак,
16 + 12 = 28 — поймал 2 рыбак,
16 + 7 = 23 — поймал 3 рыбак.
Ответ: 24, 28, 23.

Задание 6.

Имеется 8 палочек длиной в 1см, 8 палочек длиной в 2см и 7 палочек длиной в 5см.
Можно ли из всех палочек этого набора сложить прямоугольник?
Разламывать палочки нельзя.

Решение.
Если a и b – длины сторон прямоугольника, периметр P = 2(a + b),
т. е. P – четное число в случае целых a и b.
8 х 1 + 8 х 2 + 7 х 5 = 8 + 16 + 35 = 59 (см) – нечетное число.
Поэтому из всех палочек данного набора прямоугольник сложить нельзя.
Ответ: нельзя.


                     Вар-т 1            Вар-т 2            Вар-т 3

Олимпиадные задания по математике для 6 класса

Уравнения

1. Решить уравнение:
5x + 13 = 3x – 3

2. Найдите решение уравнения:
2x + 5x = –14

3. Найдите решение уравнения:
4x – 5х = 20

4. Найдите решение уравнения:
–5x + 3x = 16

5. Найдите решение уравнения:
х : 2 = –8

6. Найдите решение уравнения:
4х + 3 = 2х + 13

7. Найдите решение уравнения:
((x : 2 − 3) : 2 − 1) : 2 − 4 = 3

8. Найдите решение уравнения:
11 — 5x = 12 — 6x

9. Найдите решение уравнения:
4 • (х + 5) = 12

10. Найдите решение уравнения:
5x = 2x + 6

Задачи

Задача №1
Гравировщик делает таблички с буквами. Одинаковые буквы он гравирует за одинаковое время, разные — возможно, за разное. На две таблички «ДОМ МОДЫ» и «ВХОД» вместе он потратил 50 минут, а одну табличку «В ДЫМОХОД» сделал за 35 минут. За какое время он сделает табличку «ВЫХОД»?

Задача №2
Раньше называли число, равное миллиону миллионов , словом «легион». Если разделить миллион легионов на легион миллионов, то получится:
A) легион
B) миллион
C) миллион миллионов
D) легион легионов

Задача №3
В магазин доставили 6 бочонков с квасом, в них было 15, 16, 18, 19, 20 и 31 литр. В первый же день нашлось два покупателя: один купил два бочонка, другой – три, причем первый купил вдвое меньше кваса, чем второй. Не пришлось даже раскупоривать бочонки. Из шести бочонков на складе остался всего лишь один. Какой?

Задача №4
Молодой человек согласился работать с условием, что в конце года он получит автомобиль «Запорожец» и 2600. Но по истечении 8 месяцев уволился и при расчёте получил «Запорожец» и 1000. Сколько стоил «Запорожец»?

Задача №5
На окраску деревянного кубика затратили 4 г краски. Когда она высохла, кубик распилили на 8 одинаковых кубиков меньшего размера. Сколько краски потребуется для того, чтобы закрасить образовавшиеся при этом неокрашенные поверхности?

Задача №6
Гриша с папой ходил в тир. Уговор был такой: Гриша делает 5 выстрелов и за каждое попадание в цель получает право сделать ещё два выстрела. Всего Гриша сделал 17 выстрелов. Сколько раз Гриша попал в цель?

Задача №7
Ученик Вовочка любит решать математические задачи. Известно, что вчера он решил на 11 задач меньше, чем позавчера и на 32 задачи меньше, чем позавчера и сегодня вместе. Сколько задач решил Вовочка сегодня?

Задача №8
Чтобы сжить с белого света Змея Горыныча, которому исполнилось 40 лет, Кощей Бессмертный придумал приучить его к курению. Кощей Бессмертный подсчитал, что если Змей Горыныч каждый день в течение года будет выкуривать по 17 сигарет, то он умрет через 5 лет, если же он будет выкуривать по 16 сигарет, то умрет через 10 лет. До скольких лет доживет Змей Горыныч, если он не будет курить?

Задача №9
В затруднительном положении оказались однажды трое пеших разведчиков, которым необходимо было перебраться на противоположный берег реки при отсутствии моста. Правда, по реке катались в лодке два мальчика, готовые помочь солдатам, Но лодка была так мала, что могла выдержать вес только одного солдата; даже солдат и один мальчик не могли одновременно сесть в нее без риска ее потопить. Плавать солдаты совсем но умели. Казалось бы, при таких условиях мог переправиться через реку только один солдат. Между тем все три разведчика вскоре благополучно переправились на противоположный берег и возвратили лодку мальчикам. Как это они сделали?

Задача №10
Один из пяти братьев – Андрей, Витя, Дима, Толя или Юра разбил окно. Андрей сказал: “Это сделал или Витя, или Толя”. Витя сказал: “Это сделал не я и не Юра”. Дима сказал: “Нет, один из них сказал правду, а другой – неправду”. Юра сказал: “Нет, Дима, ты не прав”. Их отец, которому, конечно, можно доверять, уверен, что не менее трех братьев сказали правду. Кто же из братьев разбил окно?

Математические загадки

Загадка №1
У 28 человек 5 «Ы» класса на собрание пришли папы и мамы. Мам было — 24, пап — 18. У скольких учеников на собрание пришли одновременно и папа и мама?

Загадка №2
В ящике лежат 100 синих, 100 красных, 100 зелёных и 100 фиолетовых карандашей. Сколько карандашей необходимо достать, не заглядывая в ящик, чтобы среди них обязательно нашлись по крайней мере 1 красный и 1 фиолетовый.

Загадка №3
На сколько нулей оканчивается произведение 1•2•3•4•…•37?

Загадка №4
Два невисокосных года идут подряд. В первом из них больше понедельников, чем сред. Какой из семи дней чаще всего встречается во втором году?

Загадка №5
Разбейте число 186 на три попарно различных натуральных слагаемых, сумма любых двух из которых делится на третье.

Ответы к уравнениям

Уравнение

№ 1

№ 2

№ 3

№ 4

№ 5

Ответ

x = – 8

x = –2

х = 4

x = 8

х = 16

Уравнение

№ 6

№ 7

№ 8

№ 9

№ 10

Ответ

х = –5

x = 66

x = 1

х = -2

х = 2

Ответы к задачам

Задача 1
20 минут

Задача 2
Вариант А

Задача 3
Первый покупатель купил 15-литровый и 18-литровый бочонки. Второй – 16-литровый, 19-литровый и 31-литровый. Остался не проданным 20-литровый бочонок.

Задача 4
2200

Задача 5
4 грамма

Задача 6
6 раз

Задача 7
21 задачу

Задача 8
130 лет

Задача 9
9 цифр

Задача 10
Толя разбил окно

Ответы на загадки

Загадка 1
14 учеников

Загадка 2
301 карандаш

Загадка 3
8 нулей

Загадка 4
Вторник

Загадка 5
31+62+93

ОЛИМПИАДНЫЕ задания по математике для 6 класса

1. Расшифруйте числовой ребус. Одинаковым буквам соответствуют одинаковые цифры.

СЕЛ СЕЛ = ПОДСЕЛ

2. К празднику были срезаны розы: белых и розовых – 400 штук, розовых и красных – 300, белых и красных – 440. Сколько роз каждого цвета было срезано?

3. Сколько потребуется отдельных металлических цифр для нумерации 113 комнат гостиницы «Русский дом»?

4. Сейчас Коле 11 лет, а Пете 1 год. Сколько лет будет Коле и Пете, когда Коля будет втрое старше Пети?

5. В зоопарке одного города жили три кенгуру: Луиза, Дженни и Алекс. А потом родился крошка Ку. Всё это семейство съедает 28 кг моркови в неделю. Причём Ку съедает ровно вдвое меньше, чем любой из старших кенгуру. Сколько моркови в неделю съедало это семейство до рождения крошки Ку?

1) 25 кг 2) 24 кг 3) 26 кг

4) 23 кг 5) 22 кг 6) правильного ответа нет

6. На доске было написано двузначное число. Саша переставил цифры, и полученное число увеличилось в 4,5 раза. Какое двузначное число было записано первоначально?

7. За весну Винни-Пух сбавил в весе на 25%, а за лето прибавил 20%, за осень похудел на 10%, за зиму прибавил 20%. Похудел он или поправился за год?

8. Володя придумал интересный ребус. Расшифруйте его

ПЧЁЛКА · 7 = ЖЖЖЖЖЖ

9. Из двух городов, расстояние между которыми 320 км, одновременно навстречу друг другу выходят два поезда. Один идёт со скоростью 45 км/час, другой – 35 км/час. Вместе с первым поездом вылетает ласточка со скоростью 50 км/час и летит навстречу второму поезду. Встретив этот поезд, ласточка поворачивает обратно и летит навстречу первому поезду. Встретив этот поезд, она летит опять навстречу второму и так далее. Какое расстояние пролетит ласточка, пока поезда не встретятся?

1) 180 км 2) 200 км 3) 210 км

4) 140 км 5) 150 км 6) правильного ответа нет

10. Сколько процентов от часа составляют 12 минут?

1) 20% 2) 12% 3)2% 4)25%

11. При каком значении b выражение 2b+3 не является простым числом?

1) 1 2) 2 3) 3 4) 4

12. Среднее арифметическое десяти чисел равно 8, а среднее арифметическое девяти из них равно 7 . Чему равно десятое число?

1) 4 2)   3) 12 4) 9

13. 12% числа х составляют 28% числа 420. Чему равен х?

14. Какому числу кратно выражение:

4(1,5 – 3х) – 1,2(2,5 – 15х) при любом натуральном значении х?

15. Длины сторон треугольника относятся как 2 : 3 : 4. Чему равен периметр треугольника, если разность наибольшей и наименьшей сторон равна 6 см?

1) 27 2) 9 3) 18 4) 36

16. Сумма длин всех рёбер куба равна 36 см. Чему равен его объём?

1) 216 см2  2) 64 см2  3) 729 см2  4) 27 см2

17. В записи 52*2* замените звёздочки цифрами так, чтобы полученное число делилось на 36. Укажите все возможные решения.

18. Выразите число 16 с помощью четырёх пятёрок, соединяя их знаками действий.

19. Найдите два корня уравнения

| — 0,63 | : | х | = | — 0,9 |

20. В классе 35 учеников. Из них: 20 школьников занимаются в математическом кружке, 11 – в экологическом, 10 ребят не посещают эти кружки. Сколько экологов увлекается математикой?

Ответы, рекомендации, решения.

1. Решение: 625 х 625 = 390625

2. Ответ: к празднику были срезаны розы: белых – 270, розовых – 130, красных – 170.

3. Ответ: 231.

4. Решение: через Х лет Коля будет втрое старше Пети, составим уравнение 11+х=(1+х)х3, решив его получим х=4, тогда 11+4=15(лет) – Коле, 1+4=5(лет) – Пете.

5. Ответ: 2) 24 кг

6. Ответ: 18

7. Ответ: Винни — Пух похудел за год.

8. Решение: 142857х7=999999

9. Решение: 45км/ч + 35 км/ч = 80 км/ч,

320 км : 80 км/ч = 4 ч,

50 км/ч · 4 ч = 200 км

10. Ответ: 2) 20%

11. Ответ: 3) 3

12. Ответ: 3) 12

13. Решение: 0,12х=420·0,28; х=980

14. Решение: 4(1,5 – 3х) – 1,2(2,5 – 15х)=6 – 12х – 3 + 18х=3+6х=3(1+2х), данное выражение кратно 3.

15. Ответ: 1) 27

16. Решение: 36 см : 12 = 3см, V=33=27(см3)

17. Рекомендации. Число делится на 36, если оно делится и на 4 и на 9. Так как сумма цифр 5, 2, 2 равна 9, то сумма двух недостающих цифр должна равняться 0, 9 или 18. Учитывая, что число должно делиться на 4, а предпоследняя цифра равна 2, то последняя цифра может быть лишь 0 или 4 или 8. Тогда ответ будет: 52524, 52128, 52020, 52920.

18. Решение: 55 : 5 + 5 = 16

19. Ответ: х=0,7 и х=-0,7

20. Решение: 35 – 10 = 25 (уч) – посещают кружки

25 – 20 = 5 (уч) – посещают экологический кружок

11 – 5 = 6 (уч) – посещают оба кружка

Ответ: 6 экологов увлекаются математикой.

Задания Шестой Олимпиады по математике Зима 2020 6 класс

Задача №1

В слове СИСТЕМАТИКА каждую букву заменили цифрой
(Одинаковыми буквами обозначены одинаковые цифры, разными – разные)
Оказалось, что произведение всех цифр полученного числа делится нацело на 128.
Может ли это произведение оканчиваться двумя нулями?

 

Задача №2

Васе лет столько же сколько остальным трём его братьям вместе. Вася старше Миши на столько, на сколько Миша старше Олега. Петя младше Миши на 8 лет. Сколько лет Олегу?

 

Задача №3

Перед вами лежит 100 монет, выложенных по возрастанию веса от лёгкой к тяжелой.
У вас есть ещё одна монета, которая весит как одна из 100, выложенных перед вами.
Можно ли за 6 взвешиваний на чашечных весах среди 100 монет найти монету, равную вашей по весу?

 

Задача №4

Толщина корки апельсина в 9 раз меньше диаметра всего апельсина
Что больше объём мякоти апельсина или объём корки?
(Считаем, что апельсин — это идеальный шар)

 

Задача №5

Красная Шапочка вышла из дома в 9-00 и пошла к бабушке по тропинке через лес.
По дороге она иногда шла быстрее, а иногда медленнее. Иногда она делала остановку, чтобы отдохнуть. Ровно в 12-00 она пришла к бабушке. На следующий день она вышла в 9-00 и пошла домой по той же тропинке. Она опять шла с разной скоростью и иногда отдыхала. И ровно в 12-00 пришла домой.
Обязательно ли на тропинке есть такое место, в котором она была в одно и то же время в первый и во второй день?

 

Задача №6

Два катера стартуют одновременно и плывут навстречу, встречаются в 40 метрах от левого берега, разворачиваются без потерь времени и следующий раз встречаются в 30 метрах от правого берега.
Найти ширину реки.
(Скорость каждого катера всегда постоянна)

 

Задача №7

Переложи две спички, чтобы получилось верное равенство

 

Задача №8

На планете Плюк всего пять стран. Может ли быть такое, что каждая из стран имеет границу с остальными четырьмя?

Рабочие листы задач по математике для 6-го класса с ответами

Важные факты о решении и оценивании текстовых задач для 6-го класса

В значительной степени наши удивительные рабочие листы для решения проблем и оценки помогут вашим юным ученикам, изучающим математику, быстро понять важность навыков оценки в математических концепциях и реальной жизни.

Следует отметить, что эти навыки решения задач и оценки являются не только ключевой частью математических концепций, но и не менее важным методом, позволяющим похвастаться умственными математическими навыками, способностями логического и творческого мышления вашего ребенка.

Как навыки оценивания могут улучшить точность детей и повысить их знатоков математики?

Если ваши дети могут живо оценивать и разумно, то не будет сомнений, что их точность в математике повысится, в том числе и в экспертах по математике.

Более того, обладая оценочными навыками, они могут быстро определить, находится ли их ответ в разумных пределах или нет.

Учитывая, что умение оценивать повышает интеллектуальные способности ребенка к математике, ваш классник 6 сможет прийти к разумным или конкретным ответам в мгновение ока.

Самое главное, что эти навыки решения задач и оценки не только укрепят навыки детей в основных математических операциях, но и подготовят их к таким областям продвинутой математики, как вероятность, статистика, геометрия и алгебра. На этом этапе от них потребуется применить навыки логического мышления и оценки.

Как навыки оценки актуальны в нашей повседневной жизни?

Дома, на рынке, на улице или среди друзей, наша деятельность всегда будет окружена оценкой.Это правда, поскольку мы продолжаем использовать фразу «Скажем …….».

Итак, навыки решения проблем и оценки помогут вашим детям легко;

  • Смета рецептов при приготовлении, выпечке и т. Д.
  • Оцените стоимость товаров в продуктовом магазине, т. Е. Если вы хотите остаться в рамках бюджета
  • Оцените количество людей, которых вы пригласите на предстоящее мероприятие, в зависимости от имеющегося бюджета.
  • Оценивайте и умейте распоряжаться своим драгоценным временем или тратить его.Это предотвратит небрежное отвлечение внимания, а также побудит вас выполнить свою задачу.

Основные стратегии, наилучшие для решения задач оценивания слов для 6

-го класса

Наши рабочие листы задач по математике для 6 класса с ответами являются прекрасным примером для детей, чтобы овладеть жизненно важными стратегиями, которые лучше всего подходят для решения задач оценивания слов для 6 -го класса .

Каковы же тогда эти особые стратегии, которые следует учитывать при столкновении с ситуациями решения проблем и оценки?

В большинстве случаев математические задачи со словами требуют выполнения пошаговой процедуры решения .Это относится к нашему упражнению со словами из нескольких этапов. Но прежде чем мы начнем решать эти словесные проблемы, нам нужно;

  • Внимательно прочтите всю задачу дважды, чтобы лучше понять ее ключевые слова.
  • Хорошо поняв проблему, постарайтесь оценить ответ, прежде чем решать.
  • При решении покажите пошаговый расчет, сделав видимыми различные знаки операции там, где это необходимо.

Наконец, проверьте правильность своего ответа, сравнив его с тем, который вы оценили выше.

Бесплатные задания по математике для 6-го класса

Вы здесь: Главная → Рабочие листы → 6 класс

Это исчерпывающая коллекция бесплатных распечатываемых математических листов для шестого класса, организованных по таким темам, как умножение, деление, экспоненты, разрядное значение, алгебраическое мышление, десятичные дроби, единицы измерения, соотношение, процент, разложение на простые множители, GCF, LCM, дроби, целые числа и геометрия.Они генерируются случайным образом, печатаются в вашем браузере и содержат ключ ответа. Рабочие листы подходят для любой математической программы шестого класса, но особенно хорошо подходят для учебной программы IXL по математике для шестого класса.

Рабочие листы генерируются случайным образом каждый раз, когда вы нажимаете на ссылки ниже. Вы также можете получить новый, другой, просто обновив страницу в своем браузере (нажмите F5).

Вы можете распечатать их прямо из окна браузера, но сначала проверьте, как это выглядит в «Предварительном просмотре».Если рабочий лист не умещается на странице, отрегулируйте поля, верхний и нижний колонтитулы в настройках страницы вашего браузера. Другой вариант — настроить «масштаб» на 95% или 90% в предварительном просмотре печати. В некоторых браузерах и принтерах есть опция «Печатать по размеру», которая автоматически масштабирует рабочий лист по размеру области печати.

Все рабочие листы содержат ключ ответа на 2-й странице файла.

В шестом классе ученики начнут изучать начальную алгебру (порядок операций, выражения и уравнения).Они узнают о соотношениях и процентах и ​​начинают использовать целые числа. Студенты также изучают деление на множители, факторизацию, арифметику дробей и десятичную арифметику. В геометрии основное внимание уделяется площади треугольников и многоугольников и объему прямоугольных призм. Другие темы включают округление, экспоненты, GCF, LCM и единицы измерения. Обратите внимание, что эти бесплатные рабочие листы не охватывают все темы 6-го класса; в первую очередь, они не включают решение проблем.


Умножение и деление и некоторые обзоры

Длинное умножение

Длинное деление

  • 1-значный делитель, 5-значное делимое, без остатка
  • 1-значный делитель, 5-значное делимое, с остатком
  • 1-значный делитель, 6-значное делимое, без остатка
  • 1-значный делитель, 6-значное делимое, с остатком
  • 1-значный делитель, 7-значное делимое, без остатка
  • 1-значный делитель, 7-значное делимое, с остатком

  • 2-значный делитель, 5-значное делимое, без остатка
  • 2-значный делитель, 5-значное делимое, с остатком
  • 2-значный делитель, 6-значное делимое, без остатка
  • 2-значный делитель, 6-значное делимое, с остатком
  • 2-значный делитель, 7-значное делимое, без остатка
  • 2-значный делитель, 7-значное делимое, с остатком

  • 3-значный делитель, 6-значное делимое, без остатка
  • 3-значный делитель, 6-значное делимое, с остатком
  • 3-значный делитель, 7-значное делимое, без остатка
  • 3-значный делитель, 7-значное делимое, с остатком

  • Умножение десятичных знаков, запись чисел друг под другом (0-2 десятичные цифры)
  • Разделите целое число или десятичную дробь на целое число, к делимому нужно добавить нули
  • Преобразование дроби в десятичную дробь с помощью длинного деления с округлением ответов до трех знаков после запятой.

Преобразование единиц измерения с помощью деления в столбик и умножения


Математика для начальных классов Эдвард Заккаро

Хорошая книга по решению проблем с очень разнообразными текстовыми задачами и стратегиями решения проблем.Включает главы по следующим темам: последовательности, решение задач, деньги, проценты, алгебраическое мышление, отрицательные числа, логика, отношения, вероятность, измерения, дроби, деление. Вопросы в каждой главе разбиты на четыре уровня: легкий, несколько сложный, сложный и очень сложный.


Экспоненты


Разрядное значение / округление


Алгебра

Порядок работы

  • Три операции, использует ÷ для деления, без показателей
  • Четыре операции, использует ÷ для деления, без показателей
  • Две или три операции, для деления используется дробная линия, без показателей
  • Две или три операции, для деления используется дробная линия, включая показатели степени
  • Две, три или четыре операции, используется дробная линия, включая показатели степени

Выражения

Уравнения

Ключ к учебным пособиям по алгебре

Key to Algebra предлагает уникальный проверенный способ познакомить студентов с алгеброй.Новые концепции объясняются простым языком, а примеры легко следовать. Задачи со словами связывают алгебру с знакомыми ситуациями, помогая учащимся понять абстрактные концепции. Учащиеся развивают понимание, интуитивно решая уравнения и неравенства, прежде чем будут представлены формальные решения. Студенты начинают изучение алгебры с книг 1–4, используя только целые числа. Книги 5-7 вводят рациональные числа и выражения. Книги 8-10 охватывают реальную систему счисления.

=> Узнать больше


Дроби vs.Десятичные

  • Десятичные дроби или смешанные числа (десятые / сотые / тысячные)
  • Десятичные дроби или смешанные числа (с точностью до миллионной)
  • Смешанные числа с десятичными знаками (знаменатели 10, 100 и 1000)
  • Правильные и неправильные дроби с десятичными знаками (знаменатели 10, 100 или 1000)

  • Дроби правильные до десятичных (знаменатели в степени от десяти до 1000000)
  • Смешанные числа с десятичными знаками (знаменатели со степенью десяти, до 1 000 000)

  • Дроби или смешанные числа с десятичными знаками (простые, различные знаменатели)
  • Дробь в десятичную дробь — нужно деление в столбик
  • Дроби в десятичные дроби — смешанная практика

Сложение и вычитание десятичных чисел


Ключ к книгам с десятичными знаками

Это серия учебных пособий компании Key Curriculum Press, которая начинается с основных понятий и операций с десятичными знаками.Затем книги охватывают реальное использование десятичных дробей в ценообразовании, спорте, метриках, калькуляторах и науке.

В комплекте книги 1-4.

=> Узнать больше


Десятичное умножение

Умножение умственных способностей

Умножить по столбцам


Десятичное деление

Психологическое отделение

  • Простое десятичное деление (делимое состоит из 1-2 десятичных цифр, делитель целого числа)
  • То же, что и выше, но без дивиденда или делителя
  • Разделите десятичные дроби на десятичные (подумайте, сколько раз делитель вписывается в частное.)

  • Смешанные задачи умножения и деления 1 (1 десятичная цифра)

  • Разделите целые и десятичные числа на 10, 100 или 1000
  • То же, что и выше, без дивиденда или делителя
  • Умножайте или делите десятичные и целые числа на 10, 100 и 1000

  • Разделите целые и десятичные числа на 10, 100, 1000 или 10 000
  • Разделите целые и десятичные числа на 10, 100, 1000 или 10 000 — делимое или делитель отсутствует

Длинное деление


Единицы измерения

Обычная система

Преобразование единиц измерения с помощью деления в столбик и умножения (бумага и карандаш) или умственной математики

Преобразовать с помощью калькулятора с десятичными знаками

Метрическая система

  • Преобразование между мм, см и м — с использованием десятичных знаков
  • Преобразование между мм, см, м и км — с использованием десятичных знаков
  • Преобразование между мл и л и г и кг — с использованием десятичных знаков

  • Все метрические единицы, упомянутые выше — смешанная практика — с использованием десятичных знаков

  • Метрическая система: перевод единиц длины (мм, см, дм, м, плотина, гм, км)
  • Метрическая система: преобразование единиц веса (мг, cg, dg, g, dag, hg, kg)
  • Метрическая система: преобразование единиц объема (мл, кл, дл, л, дал, гл, кл)

  • Метрическая система: преобразование единиц длины, веса и объема

Коэффициент


Процент


Факторизация на простые множители, GCF и LCM


Сложение и вычитание дробей


Умножение на дроби

Во всех задачах умножения и деления дробей это помогает упростить, прежде чем умножать.


Фракционное подразделение


Преобразование дробей в смешанные числа и vv


Упрощенная дробь или эквивалентная дробь


Дроби и десятичные числа


Целые

Сетка координат

Сложение и вычитание

Сложение и вычитание целых чисел выходят за рамки Общих основных стандартов для 6-го класса, но некоторые учебные программы или стандарты могут включать их в 6-й класс.

Умножение и деление

Умножение и деление целых чисел выходят за рамки Общих основных стандартов для 6-го класса, но ссылки на рабочие листы включены сюда для полноты, поскольку некоторые учебные программы или стандарты могут включать их в 6-м классе.


Геометрия

Область — эти рабочие листы выполняются в координатной сетке.

Объем и площадь поверхности

Поскольку эти листы ниже содержат изображения различных размеров, сначала проверьте как выглядит рабочий лист в предварительном просмотре перед печатью. Если это не так подходит, вы можете либо распечатать его в масштабе (например, 90%), либо сделать еще один, обновляйте страницу рабочего листа (F5), пока не получите подходящую.


Дополнительные разделы

Пропорции


Круг



Если вы хотите иметь больший контроль над такими параметрами, как количество проблем, размер шрифта, интервал между проблемами или диапазон чисел, просто щелкните по этим ссылкам, чтобы самостоятельно использовать генераторы рабочих листов:


Задачи по математике для 6-х классов

Решение математических задач может напугать шестиклассников, но не должно.Использование нескольких простых формул и немного логики может помочь учащимся быстро вычислить ответы на кажущиеся неразрешимыми проблемы. Объясните ученикам, что вы можете узнать скорость (или скорость), с которой кто-то путешествует, если вы знаете расстояние и время, которое он проехал. И наоборот, если вы знаете скорость (скорость), с которой человек движется, а также расстояние, вы можете рассчитать время, которое он проехал. Вы просто используете базовую формулу: скорость, умноженная на время, равна расстоянию, или r * t = d (где «*» — это символ умножения.)

Приведенные ниже бесплатные распечатываемые рабочие листы связаны с такими проблемами, как эти, а также с другими важными проблемами, такими как определение наибольшего общего множителя, вычисление процентов и т. Д. Ответы для каждого рабочего листа представлены на следующем слайде сразу после каждого рабочего листа. Попросите учащихся поработать над проблемами, заполнить свои ответы в отведенных для этого пустых местах, а затем объяснить, как они могли бы прийти к решениям вопросов, по которым у них возникают трудности. Рабочие листы представляют собой отличный и простой способ быстро выполнить формирующую оценку для всего класса математики.

Рабочий лист № 1

Распечатать PDF : Рабочий лист № 1

В этом PDF-файле ваши ученики решат такие задачи, как: «Ваш брат проехал 117 миль за 2,25 часа, чтобы вернуться домой на каникулы. Какая средняя скорость, с которой он ехал?» и «У вас есть 15 ярдов ленты для ваших подарочных коробок. Каждая коробка получает одинаковое количество ленты. Сколько ленты получит каждая из ваших 20 подарочных коробок?»

Рабочий лист № 1 Решения

Решения для печати PDF : Рабочий лист №1 Решения

Чтобы решить первое уравнение на листе, используйте основную формулу: скорость, умноженная на время = расстояние, или r * t = d . В этом случае r = неизвестная переменная, t = 2,25 часа и d = 117 миль. Выделите переменную, разделив «r» с каждой стороны уравнения, чтобы получить пересмотренную формулу: r = t ÷ d . Подставьте числа, чтобы получить: r = 117 ÷ 2,25, дает r = 52 миль в час .

Для решения второй проблемы вам даже не нужно использовать формулу — просто математика и здравый смысл.Задача заключается в простом делении: 15 ярдов ленты, разделенных на 20 ящиков, можно сократить как 15 ÷ 20 = 0,75. Таким образом, каждая коробка имеет 0,75 ярда ленты.

Рабочий лист № 2

Распечатать PDF : Рабочий лист № 2

На рабочем листе № 2 учащиеся решают задачи, требующие немного логики и знания факторов, таких как: «Я думаю о двух числах, 12 и другом. У 12 и другого моего числа наибольший общий множитель равен 6, а их наименьшее общее кратное — 36.О каком еще номере я думаю? »

Другие задачи требуют только базовых знаний о процентах, а также о том, как преобразовать проценты в десятичные дроби, например: «Жасмин имеет 50 шариков в сумке. 20% шариков синие. Сколько шариков синие?»

Рабочий лист № 2 Решение

Печать решений в формате PDF : Рабочий лист № 2 Решение

Для первой задачи на этом листе вам необходимо знать, что коэффициенты из 12 равны 1, 2, 3, 4, 6 и 12 ; а , кратные 12, равны 12, 24, 36 .(Вы останавливаетесь на 36, потому что проблема говорит, что это число является наименьшим общим кратным.) Давайте выберем 6 как возможное наибольшее общее кратное, потому что это наибольший множитель 12, кроме 12. кратные 6 равны 6, 12, 18 , 24, 30 и 36 . Шесть может перейти в 36 шесть раз (6 x 6), 12 может перейти в 36 три раза (12 x 3), а 18 может перейти в 36 дважды (18 x 2), а 24 — нет. Следовательно, ответ — 18, поскольку 18 — это наибольшее общее кратное, которое может перейти в 36 .

Для второго ответа решение проще: сначала преобразуйте 20% в десятичное число, чтобы получить 0.20. Затем умножьте количество шариков (50) на 0,20. Задачу можно поставить следующим образом: 0,20 x 50 шариков = 10 синих шариков .

Задачи и решения по математическим словам

Проблема 1 Днем продавец продал в два раза больше груш, чем утром. Если он продал в тот день 360 килограммов груш, сколько? килограммов он продал утром, а сколько днем?
Нажмите, чтобы увидеть решение

Решение:
Пусть $ x $ будет килограммами, которые он продал утром.Затем днем ​​он продал по 2 доллара за килограммы. Так что итого $ x + 2x = 3x $. Это должно быть равно 360.
$ 3x = 360 $
$ x = \ frac {360} {3}
$ x = 120 $
Следовательно, продавец продал утром 120 кг и 2 \ cdot 120 = 240 $ кг днем.

Задача 2 Мэри, Питер и Люси собирали каштаны. Мэри собрала в два раза больше каштанов, чем Питер. Люси выбрала На 2 кг больше Питера. Вместе они втроем собрали 26 кг каштанов. Сколько килограммов набрал каждый из них?
Нажмите, чтобы увидеть решение

Решение:
Пусть $ x $ будет выбранной Питером суммой.Затем Мэри и Люси выбрали $ 2x $ и $ x + 2 $ соответственно. Итак,
$ x + 2x + x + 2 = 26
$ 4x = 24
$ x = 6
$ Таким образом, Питер, Мэри и Люси выбрали 6, 12 и 8 кг соответственно.

Задача 3
София закончила $ \ frac {2} {3} $ книги. Она подсчитала, что закончила на 90 страниц больше, чем еще не прочитала. Как долго ее книга?
Нажмите, чтобы увидеть решение

Решение:
Пусть $ x $ будет общим количеством страниц в книге, тогда она закончила $ \ frac {2} {3} \ cdot x $ страниц.
Тогда у нее осталось $ x- \ frac {2} {3} \ cdot x = \ frac {1} {3} \ cdot x $ страниц.
$ \ frac {2} {3} \ cdot x- \ frac {1} {3} \ cdot x = 90 $
$ \ frac {1} {3} \ cdot x = 90 $
$ x = 270 $
Итак, в книге 270 страниц.

Задача 4
Сельскохозяйственное поле можно обработать 6 тракторами за 4 дня. Когда 6 тракторов работают вместе, каждый из них пашет. 120 га в сутки. Если два трактора были перенесены на другое поле, тогда оставшиеся 4 трактора могут вспахать то же поле за 5 дней.Сколько гектаров в день будет обрабатывать один трактор?
Нажмите, чтобы увидеть решение

Решение:
Если каждый из тракторов за 6 долларов обрабатывает 120 долларов гектаров в день, и они заканчивают работу за 4 доллара. дней, то все поле будет: 120 $ \ cdot 6 \ cdot 4 = 720 \ cdot 4 = 2880 $ га. Давайте предположим, что каждый из четырех тракторов обрабатывал $ x $ гектаров в день. Таким образом, за 5 дней вспахано
$ 5 \ cdot 4 \ cdot x = 20 \ cdot x $ га, что равняется площади всего поля, 2880 га.
Итак, получаем $ 20x = 2880 $
$ x = \ frac {2880} {20} = 144 $. Таким образом, каждый из четырех тракторов будет обрабатывать 144 гектара в день.

Задача 5
Студент выбрал число, умножил его на 2, затем вычел 138 из результата и получил 102. Какое число он выбрал?
Нажмите, чтобы увидеть решение

Решение:
Пусть $ x $ будет выбранным им числом, тогда
$ 2 \ cdot x — 138 = 102 $
$ 2x = 240 $
$ x = 120 $

Задача 6
Я выбрал число и разделил его на 5.Затем я вычел из результата 154 и получил 6. Какое число я выбрал?
Нажмите, чтобы увидеть решение

Решение:
Пусть $ x $ будет выбранным мной числом, тогда
$ \ frac {x} {5} -154 = 6 $
$ \ frac {x} {5} = 160 $ ​​
$ x = 800 $

Задача 7
Расстояние между двумя городами 380 км. В этот же момент легковой автомобиль и грузовик начинают движение навстречу друг другу из разные города. Они встречаются через 4 часа. Если автомобиль движется на 5 км / ч быстрее грузовика, какова их скорость?
Нажмите, чтобы увидеть решение

Решение:
Основная идея, используемая в такого рода задачах, заключается в том, что расстояние равно скорости, умноженной на время $ S = V \ cdot t $.
В (км / ч) т (час) S (км)
Автомобиль х + 5 4 4 (х +5)
Грузовик Х 4 4x
4 (x + 5) + 4x = 380
$ 4x + 4x = 380 — 20
$ 8x = 360
$ x = \ frac {360} {8}
$ x = 45
$ Следовательно, скорость грузовика составляет 45 долларов за км / час, а скорость автомобиля — 50 долларов за км / час.

Задача 8
Одна сторона прямоугольника на 3 см короче другой стороны. Если увеличить длину каждой стороны на 1 см, то площадь прямоугольника увеличится на 18 см 2 . Найдите длины всех сторон.
Нажмите, чтобы увидеть решение

Решение:
Пусть $ x $ будет длиной большей стороны $ x \ gt 3 $, тогда длина другой стороны будет $ x-3 $ см. Тогда площадь S 1 = x (x — 3) см 2 . После увеличения длины сторон они станут $ (x +1) $ и $ (x — 3 + 1) = (x — 2) $ см в длину.2 + x — 2x — 2
$ 2x = 20
$ x = 10 $. Итак, стороны прямоугольника равны $ 10 $ см и $ (10 — 3) = 7 $ см в длину.

Задача 9
В первый год две коровы дали 8100 литров молока. На второй год их производство увеличилось. на 15% и 10% соответственно, а общее количество молока увеличилось до 9100 литров в год. Сколько литров молока давалось от каждой коровы за год?
Нажмите, чтобы увидеть решение

Решение:
Пусть x будет количеством молока первой коровы произведен в течение первого года.Затем вторая корова в тот год произвела (8100 — x) литров молока. На второй год каждая корова произвела такое же количество молока, как и в первый год, плюс увеличение на 15 \% $ или 10 \% $.
Итак, 8100 $ + \ frac {15} {100} \ cdot x + \ frac {10} {100} \ cdot (8100 — x) = 9100 $
Следовательно, 8100 $ + \ frac {3} {20} x + \ frac {1} {10} (8100 — x) = 9100 $
$ \ frac {1} {20} x = 190 $
$ x = 3800 $
Следовательно, коровы дали 3800 и 4300 литров молока в первый год и 4370 долларов и 4730 долларов за литр молока во второй год, соответственно.

Проблема 10
расстояние между станциями A и B — 148 км. Экспресс отправился со станции A в сторону станции B со скоростью 80 км / ч. В то же В это время товарный поезд покинул станцию ​​B в сторону станции A со скоростью 36 км / час. Они встретились на станции C в 12 часов, и к тому времени экспресс остановился на промежуточной станции на 10 мин, а грузовой поезд остановился на 5 мин. Найдите:
а) Расстояние между станциями C и B.
б) Время, когда грузовой поезд покинул станцию ​​B.
Нажмите, чтобы увидеть решение

Решение
a) Пусть x будет расстоянием между станции B и C. Тогда расстояние от станции C до станции A составляет $ (148 — x) $ км. К моменту встречи на станции C экспресс ехал $ \ frac {148-x} {80} + \ frac {10} {60} $ часов, а грузовой поезд ехал $ \ frac {x} {36} + \ frac {5} {60} $ часов . Поезда ушли одновременно, так что: $ \ frac {148 — x} {80} + \ frac {1} {6} = \ frac {x} {36} + \ frac {1} {12} $. Общий знаменатель чисел 6, 12, 36, 80 равен 720.Тогда
$ 9 (148 — x) +120 = 20x + 60 $
$ 1332 — 9x + 120 = 20x + 60 $
$ 29x = 1392 $
$ x = 48 $. Таким образом, расстояние между станциями B и C составляет 48 км.
б) К моменту встречи на станции С фрахт поезд ехал $ \ frac {48} {36} + \ frac {5} {60} $ часов, то есть 1 доллар в час и 25 долларов в минуту.
Следовательно, он покинул станцию ​​B на отметке $ 12 — (1 + \ frac {25} {60}) = 10 + \ frac {35} {60} $ часов, то есть в 10:35.

Задача 11
Сьюзен едет из города А в город Б.После двух часов езды она заметил, что она преодолела 80 км и подсчитал, что если она продолжит двигаясь с той же скоростью, она опаздывала на 15 минут. Так она увеличила скорость на 10 км / ч и прибыла в город B на 36 минут раньше чем она планировала.
Найдите расстояние между городами A и B.
Нажмите, чтобы увидеть решение

Решение:
Пусть $ x $ будет расстоянием между точками A и B. Поскольку Сьюзен преодолела 80 км за 2 часа, ее скорость составила $ V = \ frac {80} {2} = 40 $ км / час.
Если бы она продолжила движение с той же скоростью, то опоздала бы на 15 $ минут, т.е. запланированное время в пути составляет $ \ frac {x} {40} — \ frac {15} {60} $ hr. Остальное расстояние $ (x — 80) $ км. $ V = 40 + 10 = 50 $ км / час.
Итак, она преодолела расстояние между A и B за $ 2 + \ frac {x — 80} {50} $ час, и это оказалось на 36 минут меньше, чем планировалось. Таким образом, запланированное время было $ 2 + \ frac {x -80} {50} + \ frac {36} {60} $.
Когда мы выравниваем выражения для запланированного времени, мы получаем уравнение:
$ \ frac {x} {40} — \ frac {15} {60} = 2 + \ frac {x -80} {50} + \ frac {36} {60} $
$ \ frac {x — 10} {40} = \ frac {100 + x — 80 + 30} {50} $
$ \ frac {x — 10} {4} = \ frac {x +50} {5} $
$ 5x — 50 = 4x + 200 $
$ x = 250 $
Итак, расстояние между городами A и B составляет 250 км.

Задача 12
Чтобы доставить заказ вовремя, компания должна производить 25 деталей в день. После изготовления 25 частей в день по 3 дней компания начала производить на 5 деталей больше в день, а к последнему дню работы было произведено на 100 деталей больше, чем планировалось. Узнайте, сколько деталей изготовила компания и сколько дней это заняло.
Нажмите, чтобы увидеть решение

Решение:
Пусть $ x $ будет количеством дней, в течение которых компания проработала. Тогда 25x — это количество деталей, которые они планировали изготовить.При новом уровне добычи они сделано:
$ 3 \ cdot 25 + (x — 3) \ cdot 30 = 75 + 30 (x — 3) $
Следовательно: 25 $ x = 75 + 30 (x -3) — 100 $
$ 25x = 75 + 30x -90 — 100 $
$ 190 -75 = 30x -25 $
$ 115 = 5x
$ x = 23
$ Таким образом, компания проработала 23 дня, и они заработали 23 $ \ cdot 25 + 100 = 675 $ штук.

Задача 13
В седьмом классе 24 ученика. Решили посадить на заднем дворе школы березы и розы. Пока каждая девочка посадила по 3 роз, каждые три мальчика посадили по 1 берёзе.К концу дня они посадили растения за 24 доллара. Сколько берез и роз было посажено?
Нажмите, чтобы увидеть решение

Решение:
Пусть $ x $ будет количеством роз. Тогда количество берез составляет 24 $ — x $, а количество мальчиков — $ 3 \ times (24-x) $. Если каждая девочка посадила 3 роз, в классе $ \ frac {x} {3} $ девочек.
Мы знаем, что в классе 24 ученика. Следовательно, $ \ frac {x} {3} + 3 (24 — x) = 24 $
$ x + 9 (24 — x) = 3 \ cdot 24 $
$ x +216 — 9x = 72 $
$ 216 — 72 = 8x $
$ \ frac {144} {8} = x
$ $ x = 18
$ Итак, ученики посадили 18 роз и 24 — x = 24 — 18 = 6 берез.

Задача 14
Автомобиль выехал из города A в сторону города B, двигаясь со скоростью V = 32 км / час. После 3 часов в пути водитель остановился на 15 минут в городе C. на закрытой дороге ему пришлось изменить маршрут, увеличив поездку на 28 км. Он увеличил скорость до V = 40 км / час, но все равно опоздал на 30 минут. Находим:
а) Расстояние, которое преодолела машина.
b) Время, которое потребовалось, чтобы добраться от C до B.
Щелкните, чтобы увидеть решение

Решение:
Из постановки задачи мы не знаем, была ли 15-минутная остановка в городе C запланирована или она была запланирована. непредвиденный.Итак, мы должны рассмотреть оба случая.

A
Остановка была запланирована. Рассмотрим только поездку из C в B, и пусть $ x $ будет количеством часов, в течение которых водитель потратил на эту поездку.
Тогда расстояние от C до B равно $ S = 40 \ cdot x $ км. Если бы водитель мог использовать первоначальный маршрут, ему потребовалось бы $ x — \ frac {30} {60} = x — \ frac {1} {2} $ часов, чтобы проехать от C до B. Расстояние от C до B. согласно первоначальному маршруту $ (x — \ frac {1} {2}) \ cdot 32 $ км, и это расстояние на $ 28 $ км короче, чем $ 40 \ cdot x $ км.Тогда у нас есть уравнение
$ (x — 1/2) \ cdot 32 + 28 = 40x $
$ 32x -16 +28 = 40x $
$ -8x = -12 $
$ 8x = 12 $.
$ x = \ frac {12} {8} $
$ x = 1 \ frac {4} {8} = 1 \ frac {1} {2} = 1 \ frac {30} {60} = 1 час. 30 минут.
Итак, автомобиль преодолел расстояние от C до B за 1 час 30 минут.
Расстояние от A до B составляет $ 3 \ cdot 32 + \ frac {12} {8} \ cdot 40 = 96 + 60 = 156 $ км.

B
Предположим, ему потребовалось $ x $ часов чтобы добраться из C в B. Тогда расстояние $ S = 40 \ cdot x $ км.
Водитель не планировал остановку на C. Допустим, он остановился, потому что ему пришлось изменить маршрут.
Потребовалось $ x — \ frac {30} {60} + \ frac {15} {60} = x — \ frac {15} {60} = x — \ frac {1} {4} $ h, чтобы проехать от С к Б. расстояние от C до B составляет 32 (x — \ frac {1} {4}) $ км, что на 28 $ км короче, чем $ 40 \ cdot x $, т.е.
$ 32 (x — \ frac {1} {4}) + 28 = 40x $
$ 32x — 8 +28 = 40x $
$ 20 = 8x $
$ x = \ frac {20} {8} = \ frac {5} {2} = 2 \ text {hr} 30 \ text {min}. $
Пройденное расстояние равно $ 40 \ times 2.5 = 100 км $.

Задача 15
Если фермер хочет вовремя вспахивать поле фермы, он должен вспахивать 120 гектаров в день. По техническим причинам он пахал всего 85 гектаров в день, следовательно, ему пришлось пахать на 2 дня больше, чем планировалось, и он осталось еще 40 га. Какова площадь фермерского поля и сколько дней фермер изначально планировал работать?
Нажмите, чтобы увидеть решение

Решение:
Пусть $ x $ будет количеством дней в первоначальном плане.Следовательно, все поле составляет 120 \ cdot x $ га. Фермеру приходилось работать x + 2 доллара в день, и он вспахали 85 долларов (x + 2) гектаров, оставив 40 гектаров невыпаханными. Тогда у нас есть уравнение:
$ 120x = 85 (x + 2) + 40 $
$ 35x = 210 $
$ x = 6 $.
Итак, фермер планировал завершить работы за 6 дней, а площадь фермерского поля составляет 120 $ \ cdot 6 = 720 $ га.

Задача 16
Столяр обычно делает определенное количество запчасти за 24 дня. Но он смог увеличить свою производительность на 5 деталей в день, и поэтому он не только закончил работу всего за 22 дня, но и сделал 80 дополнительных деталей.Сколько частей плотник обычно делает в день, а сколько штук он делает за 24 дня?
Нажмите, чтобы увидеть решение

Решение:
Пусть $ x $ будет количеством деталей, которые плотник обычно изготавливает ежедневно. За 24 дня он заработал $ 24 \ cdot x $ штук. Его новая дневная норма производства составляет x + 5 долларов за штуку и в $ 22 $ дней он сделал $ 22 \ cdot (x + 5) $ деталей. Это на 80 больше, чем $ 24 \ cdot x $. Следовательно уравнение:
$ 24 \ cdot x + 80 = 22 (x +5) $
$ 30 = 2x
$ x = 15 $
Обычно он делает 15 деталей в день, а за 24 дня он зарабатывает 15 $ \ cdot 24 = 360 $ частей.

Задача 17
Байкер преодолел половину расстояния между двумя городами за 2 часа 30 минут. После этого он увеличил скорость на 2 км / час. Вторую половину дистанции он преодолел за 2 часа 20 минут. Найдите расстояние между двумя городами и начальная скорость байкера.
Нажмите, чтобы увидеть решение

Решение:
Пусть x км / ч будет начальной скоростью байкером, то его скорость во второй части поездки составляет x + 2 км / час. Половина расстояния между двумя городами равна $ 2 \ frac {30} {60} \ cdot x $ км и $ 2 \ frac {20} {60} \ cdot (x + 2) $ км.Из уравнения: $ 2 \ frac {30} {60} \ cdot x = 2 \ frac {20} {60} \ cdot (x + 2) $ получаем $ x = 28 $ км / час.
Начальная скорость байкера 28 км / ч.
Половина расстояния между двумя городами составляет
$ 2 ч 30 мин \ раз 28 = 2,5 \ раз 28 = 70 $.
Итак, расстояние 2 $ \ умноженное на 70 = 140 $ км.

Задача 18
Поезд преодолел половину расстояния между станциями A и B со скоростью 48 км / час, но затем ему пришлось остановиться на 15 мин. Составить из-за задержки он увеличил свою скорость на $ \ frac {5} {3} $ м / сек и прибыл на станцию ​​B вовремя.Найдите расстояние между двумя станциями и скорость поезда после остановки.
Нажмите, чтобы увидеть решение

Решение:
Сначала определим скорость поезда после остановки. Скорость было увеличено на $ \ frac {5} {3} $ м / сек $ = \ frac {5 \ cdot 60 \ cdot 60} {\ frac {3} {1000}} $ км / час = $ 6 $ км / час. Следовательно новая скорость 48 $ + 6 = 54 $ км / час. Если на покрытие первого половины расстояния, то на преодоление вторая часть.
Итак, уравнение: $ 48 \ cdot x = 54 \ cdot (x — 0,25) $
$ 48 \ cdot x = 54 \ cdot x — 54 \ cdot 0,25 $
$ 48 \ cdot x — 54 \ cdot x = — 13,5 $
$ -6x = — 13,5 $
$ x = 2,25 $ ч.
Все расстояние
$ 2 \ умножить на 48 \ умножить на 2,25 = 216 $ км.

Задача 19
Элизабет может выполнить определенную работу за 15 дней, а Тони — только 75%. эта работа в одно и то же время. Тони работал один в течение нескольких дней, а затем к нему присоединилась Элизабет, так что они закончили остаток работы. работа за 6 дней, работаем вместе.
Сколько дней проработал каждый из них и какой процент работы каждый из них выполнил?
Нажмите, чтобы увидеть решение

Решение:
Сначала мы найдем дневную производительность каждого рабочего. Если мы рассмотрим всю работу как единицу (1), Элизабет выполняет $ \ frac {1} {15} $ работы в день, а Тони выполняет 75 \% $ из $ \ frac {1} {15} $, т.е.
$ \ frac { 75} {100} \ cdot \ frac {1} {15} = \ frac {1} {20} $. Предположим, что Тони работал один за $ x $ дней. Затем он в одиночку выполнил $ \ frac {x} {20} $ всей работы.Работающий вместе в течение 6 дней двое рабочих закончили $ 6 \ cdot (\ frac {1} {15} + \ frac {1} {20}) = 6 \ cdot \ frac {7} {60} = \ frac {7} { 10} $ работы.
Сумма $ \ frac {x} {20} $ и $ \ frac {7} {10} $ дает нам всю работу, т.е. $ 1 $. Получаем уравнение:
$ \ frac {x} {20} + \ frac {7} {10} = 1 $
$ \ frac {x} {20} = \ frac {3} {10} $
$ х = 6 $. Тони проработал 6 + 6 = 12 дней и Элизабет работала за 6 долларов в день. Часть работы сделана это $ 12 \ cdot \ frac {1} {20} = \ frac {60} {100} = 60 \% $ для Тони и $ 6 \ cdot \ frac {1} {15} = \ frac {40} {100} = 40 \% $ для Элизабет.

Задача 20
Фермер планировал вспахать поле, выполнив 120 га в сутки. После двух дней работы он увеличил свою дневную производительность на 25% и закончил работу на два дня раньше срока.
а) Какова площадь поля?
б) За сколько дней фермер выполнил свою работу?
c) Через сколько дней фермер планировал завершить работу?
Нажмите, чтобы увидеть решение

Решение:
Прежде всего мы найдем новую суточную производительность фермер в гектарах в сутки: 25% от 120 гектаров $ \ frac {25} {100} \ cdot 120 = 30 $ га, следовательно, 120 $ + 30 = 150 $ га новая ежедневная производительность.Пусть x будет запланированным количеством дней, отведенных на работу. Тогда хозяйство будет 120 \ cdot x $ га. На с другой стороны, мы получим ту же площадь, если добавим 120 $ \ cdot 2 $ гектаров к 150 $ (х -4) $ га. Тогда мы получаем уравнение:
$ 120x = 120 \ cdot 2 + 150 (x -4) $
$ x = 12 $
Итак, изначально предполагалось, что работа займет 12 дней, но на самом деле поле было вспахано за 12-2 дней. = 10 дней. Площадь поля 120 $ \ cdot 12 = 1440 $ га.

Задача 21
Чтобы покосить травяное поле, бригада косилок планировала обрабатывать 15 гектаров в день.Через 4 рабочих дня они увеличили дневную производительность на $ 33 \ times \ frac {1} {3} \% $ и закончил работу на 1 день раньше запланированного срока.
А) Какова площадь травяного поля?
B) Сколько дней понадобилось, чтобы косить все поле?
C) Сколько дней изначально было запланировано для этой работы?
Подсказка : Взгляните на проблему 20 и решите ее сами.
Ответ: А) 120 га; Б) 7 дней; В) 8 дней.

Задача 22
Поезд идет от станции A до станции B.Если поезд отправляется со станции А со скоростью 75 км / час, прибывает на станцию ​​B на 48 минут раньше запланированного. Если бы он двигался со скоростью 50 км / час, то к запланированному времени прибытия бы осталось еще 40 км до станции B. Найти:
A) Расстояние между двумя станциями;
B) Время, необходимое поезду, чтобы добраться из пункта А в пункт Б по расписанию;
C) Скорость поезда по расписанию.
Нажмите, чтобы увидеть решение

Решение:
Пусть $ x $ будет запланированным временем поездки из пункта А в пункт Б.Тогда расстояние между A и B можно найти двумя способами. С одной стороны, это расстояние составляет $ 75 (x — \ frac {48} {60}) $ км. С другой стороны, это 50 $ + 40 $ км. Таким образом, мы получаем уравнение:
$ 75 (x — \ frac {48} {60}) = 50x + 40 $
$ x = 4 $ час — это запланированное время в пути. В расстояние между двумя станциями составляет 50 $ \ cdot 4 + 40 = 240 $ км. Тогда скорость, которую поезд должен поддерживать, чтобы идти по расписанию, составляет $ \ frac {240} {4} = 60 $ км / час.

Задача 23
Расстояние между городами A и B составляет 300 км.Один поезд отправляется из города А, а другой — из города. город B, оба уезжают в один и тот же момент времени и направляются друг к другу. Мы знаем, что один из них на 10 км / час быстрее другого. Находить скорости обоих поездов, если через 2 часа после отправления расстояние между ними составляет 40 км.
Нажмите, чтобы увидеть решение

Решение:
Пусть скорость более медленного поезда будет $ x $ км / час. Тогда скорость более быстрый поезд стоит (x + 10) $ км / час. За 2 часа они преодолевают 2x $ км и 2 (x +10) $ км соответственно.Поэтому, если они еще не встретились, весь расстояние от A до B составляет $ 2x + 2 (x +10) +40 = 4x + 60 $ км. Однако если они уже встретились и продолжили движение, расстояние будет $ 2x + 2 (x + 10) — 40 = 4x — 20 $ км. Таким образом, мы получаем следующие уравнения:
$ 4x + 60 = 300 $
$ 4x = 240 $
$ x = 60 $ или
$ 4x — 20 = 300 $
$ 4x = 320 $
$ x = 80 $
Отсюда скорость более медленного поезда составляет 60 долларов США км / час или 80 долларов США км / час, а скорость поезда более быстрый поезд стоит 70 долларов за км / час или 90 долларов за км / час.

Задача 24
Автобус едет из города А в город Б.Если скорость автобуса составляет 50 км / час, он прибудет в город B на 42 минуты позже запланированного срока. Если автобус увеличивается его скорость составляет $ \ frac {50} {9} $ м / сек, он прибудет в город B на 30 минут раньше запланированного срока. Находим:
A) Расстояние между двумя городами;
B) Планируемое время прибытия автобуса в B;
C) Скорость автобуса по расписанию.
Нажмите, чтобы увидеть решение

Решение:
Сначала определим скорость автобуса после ее увеличения. Скорость увеличено на $ \ frac {50} {9} $ м / сек $ = \ frac {50 \ cdot60 \ cdot60} {\ frac {9} {1000}} $ км / час $ = 20 $ км / час.Следовательно, новая скорость составляет $ V = 50 + 20 = 70 $ км / час. Если $ x $ — количество часов по расписанию, то со скоростью 50 км / ч автобус едет из пункта A в пункт B за $ (x + \ frac {42} {60}) $ час. Когда скорость автобуса составляет $ V = 70 $ км / час, время в пути составляет $ x — \ frac {30} {60} $ час. потом
$ 50 (x + \ frac {42} {60}) = 70 (x- \ frac {30} {60}) $
$ 5 (x + \ frac {7} {10}) = 7 (x- \ frac { 1} {2}) $
$ \ frac {7} {2} + \ frac {7} {2} = 7x -5x $
$ 2x = 7 $
$ x = \ frac {7} {2} $ час.
Итак, автобус должен сделать поездку за 3 доллара за час 30 долларов за минуту.
Расстояние между двумя городами составляет $ 70 (\ frac {7} {2} — \ frac {1} {2}) = 70 \ cdot 3 = 210 $ км, а запланированная скорость составляет $ \ frac {210} {\ гидроразрыв {7} {2}} = 60 $ км / час.

Задания по математике для 6-х классов

Положитесь на наши распечатываемые рабочие листы по математике для 6-го класса с ключами ответов для тщательной практики. С цепочками, взятыми из жизненно важных математических тем, таких как соотношение, умножение, деление, дроби, общие множители и кратные, рациональные числа, алгебраические выражения, целые числа, одношаговые уравнения, упорядоченные пары в четырех квадрантах и ​​навыки геометрии, такие как определение площади, площади поверхности , и объем, систематизируя числовые данные в виде точечных диаграмм, гистограмм и коробчатых диаграмм, находя среднее, медианное значение и квартили наборов данных, эти PDF-файлы становятся вашим часто используемым местом.Получите свой билет к отличному старту с нашими бесплатными рабочими листами по математике для учащихся 6-х классов!

Выбор заданий по математике для 6-го класса по теме

Изучите 4600+ заданий по математике для шестого класса

Дивизия | 4 цифры по 2 и 3 цифры

Разделите 4-значное число на 2-значное или 3-значное число, начиная с наибольшего разряда и до последней цифры, чтобы найти частное и остаток (если есть).

Рисование фигур для представления соотношения

Развивайте свои навыки соотношения с помощью нашей таблицы в формате pdf по математике для 6-го класса.Предложите учащимся визуально представить условия каждого соотношения, нарисовав определенное количество фигур.

Умножение двух дробей перекрестным сокращением

Разбиение числителя и знаменателя на простые множители, взаимное сокращение общих множителей и умножение неотмененных частей дроби — все это должны делать ученики 6 класса!

Выражение десятичных знаков в словах | До миллионных

Позвольте вашему пониманию десятичных разрядов взлететь на новый уровень, когда вы погрузитесь в эти печатные математические рабочие листы для 6-го класса и выразите десятичные дроби с точностью до миллионной в словах и наоборот.

Нахождение GCF двух чисел

Следуйте пошаговому процессу нахождения GCF, перечислив все факторы каждой пары двузначных чисел, затем сравните факторы, определите общие, и самый большой из них — ваш ответ.

Представление целых чисел

Создание прочного фундамента в целых числах становится легким благодаря прикосновению к реальному миру, предлагаемому в этих таблицах в формате pdf по математике для 6-го класса, где каждый сценарий должен быть представлен положительным или отрицательным целым числом.

Выявление упорядоченных пар во всех квадрантах

Ознакомьтесь с координатной плоскостью, четырьмя квадрантами и упорядоченными парами. Определите точку, представляющую каждую упорядоченную пару в Части A; и найдите и запишите координаты x-y каждой точки в части B.

Определение площади квадрата

Предоставьте детям 6-го класса эти распечатанные рабочие листы по математике, если вы хотите попрактиковаться в определении площади квадрата. Попросите учащихся возвести в квадрат дробные длины сторон и вычислить площадь.

Идентификация сетей трехмерных фигур

Предложите учащимся 6-го класса представить, как складывать двумерные сети, представленные в этом печатном математическом листе, по краям, и визуализировать трехмерные формы, которые можно построить из них.

Площадь поверхности с использованием сетей | Подсчет квадратов

Эти PDF-файлы служат цели, будь то уточнение или проверка навыков подсчета квадратов для определения площади поверхности прямоугольных призм, кубов или треугольных призм по их сеткам.

Сводка из пяти цифр

Описательно проанализируйте каждый набор данных, переставив значения в порядке возрастания и определив максимальное и минимальное значения, а также нижний или 1-й, медианный или 2-й, и верхний или 3-й квартили.

Проблемы со словом Ratio | По частям

Проверьте свои аналитические способности с помощью нашей таблицы в формате pdf по математике для 6-го класса. Изучите данные, представленные в пиктограмме, и ответьте на ряд вопросов, определяя соотношение частей к частям в каждом сценарии.

Умножение смешанных чисел и дробей

Легко умножайте смешанные числа и дроби, преобразуя смешанные числа в неправильные дроби, взаимно сокращая общие множители и затем умножая оставшееся.

120 Задачи по математике для учащихся 1–8 классов

Вы сидите за партой, готовые вместе выполнить викторину, тест или задание по математике. Вопросы перетекают в документ до тех пор, пока вы не попадете в раздел с текстовыми проблемами.

Помогла бы толчок творчества. Но этого не произошло.

Независимо от того, являетесь ли вы учителем 3-го класса или учителем 8-го класса, готовящим учеников к старшей школе, воплощение математических концепций в примеры из реального мира, безусловно, может быть проблемой.

Этот ресурс — ваш творческий заряд. Он предоставляет примеры и шаблоны математических задач на слова для 1-8 классов.

Всего 120 примеров. Помогая вам разобраться в них, чтобы найти вопросы для ваших учеников, ресурс разделен на категории по следующим навыкам с некоторым перекрытием между темами:

Список примеров дополнен советами по созданию увлекательных и сложных математических задач со словами.

120 Математических задач со словами, классифицированных по навыкам

Сложные задачи со словами

Подходит для: 1-го класса, 2-го класса

1. Добавление к 10: Ариэль играла в баскетбол. 1 из ее выстрелов попал в обруч. 2 ее выстрела не попали в обруч. Сколько всего было выстрелов?

2. Добавление к 20: У Адрианны есть 10 кусочков жевательной резинки, которыми она может поделиться со своими друзьями. На всех ее подруг не хватило жевательной резинки, поэтому она пошла в магазин за еще тремя кусочками жевательной резинки.Сколько кусочков жевательной резинки сейчас у Адрианны?

3. Добавление к 100: У Адрианны есть 10 кусочков жевательной резинки, которыми она может поделиться со своими друзьями. На всех ее подруг не хватило жевательной резинки, поэтому она пошла в магазин и купила 70 кусочков клубничной жевательной резинки и 10 кусочков жевательной резинки. Сколько кусочков жевательной резинки сейчас у Адрианны?

4. Добавление чуть больше 100: В ресторане 175 обычных стульев и 20 стульев для младенцев. Сколько всего стульев в ресторане?

5.Добавляем к 1000: Сколько печенья вы продали, если продали 320 шоколадных печений и 270 ванильных печений?

6. Добавление до 10 000 и более: Обычно магазин товаров для хобби продает 10 576 торговых карточек в месяц. В июне в магазине товаров для хобби было продано на 15 498 карточек больше, чем обычно. В целом, сколько коллекционных карточек было продано в магазине для хобби в июне?

7. Сложение 3 чисел: У Билли дома было 2 книги. Он пошел в библиотеку, чтобы достать еще 2 книги. Затем он купил 1 книгу.Сколько книг у Билли сейчас?

8. Добавление трех чисел к 100 и более: Эшли купила большой мешок конфет. В сумке было 102 синих конфеты, 100 красных и 94 зеленых. Сколько всего было конфет?

Задачи на вычитание слов

Подходит для: 1-й сорт, 2-й сорт

9. Вычитая до 10: Всего в пиццерии было 3 пиццы. Покупатель купил 1 пиццу. Сколько пиццы осталось?

10.Вычитаем до 20: Ваша подруга сказала, что у нее 11 наклеек. Когда вы помогли ей убрать стол, у нее было всего 10 наклеек. Сколько наклеек не хватает?

11. Вычитая до 100: У Адрианны есть 100 кусочков жевательной резинки, которыми она может поделиться с друзьями. Когда она пошла в парк, она разделила 10 кусочков клубничной жевательной резинки. Когда она вышла из парка, Адрианна поделилась еще 10 кусочками жевательной резинки. Сколько кусочков жевательной резинки сейчас у Адрианны?

Зарегистрируйтесь сейчас

12. Вычитание Немного больше 100: Ваша команда набрала 123 очка.В первом тайме было набрано 67 очков. Сколько было забито во втором тайме?

13. Вычитая до 1000: У Натана большая муравьиная ферма. Он решил продать несколько своих муравьев. Он начал с 965 муравьев. Продал 213. Сколько муравьев у него сейчас?

14. Вычитая до 10 000 и более: Обычно магазин товаров для хобби продает 10 576 торговых карточек в месяц. В июле в магазине товаров для хобби было продано 20 777 коллекционных карточек. Сколько коллекционных карточек было продано в магазине в июле по сравнению с обычным месяцем?

15.Вычитая 3 числа: У Шарлин была пачка из 35 карандашей. 6 она отдала своей подруге Терезе. Она дала 3 своей подруге Мэнди. Сколько мелков осталось у Шарлин?

16. Вычитание трех чисел из 100: Эшли купила большой мешок конфет, чтобы поделиться с друзьями. Всего конфет было 296 штук. Она подарила Мариссе 105 конфет. Еще она подарила Кайле 86 конфет. Сколько конфет осталось?

Задачи умножения слов

Подходит для: 2-й класс, 3-й класс

17.Умножение однозначных целых чисел: Адрианне нужно разрезать сковороду с пирожными на кусочки. Она нарезает на сковороду 6 ровных столбиков и 3 ровных ряда. Сколько у нее пирожных?

18. Умножение 2-значных целых чисел: В кинотеатре 25 рядов сидений по 20 мест в каждом ряду. Сколько всего мест?

19. Умножение целых чисел, заканчивающееся на 0: Компания по производству одежды предлагает 4 различных вида толстовок. Ежегодно компания производит 60 000 толстовок каждого вида.Сколько свитшотов компания производит каждый год?

20. Умножение 3 целых чисел: Каменщик укладывает кирпичи в 2 ряда по 10 кирпичей в каждом ряду. Сверху каждого ряда находится стопка из 6 кирпичей. Сколько всего кирпичей?

21. Умножение 4 целых чисел: Кэли зарабатывает 5 долларов в час, разнося газеты. Она доставляет газеты 3 дня в неделю по 4 часа за раз. Сколько денег заработает Кэли после доставки газет в течение 8 недель?

Проблемы с разделением слов

Подходит для: 3-й класс, 4-й класс, 5-й класс

22.Деление однозначных целых чисел: Если у вас есть 4 конфеты, равномерно разделенных на 2 пакета, сколько конфет находится в каждом пакете?

23. Разделение двухзначных целых чисел: Если у вас есть 80 билетов на ярмарку, и каждая поездка стоит 5 билетов, сколько поездок вы сможете совершить?

24. Разделительные числа, оканчивающиеся на 0: У школы есть 20 000 долларов на покупку нового компьютерного оборудования. Если каждая единица оборудования стоит 50 долларов, сколько всего ее может купить школа?

25.Деление 3 целых чисел: Мелисса покупает 2 упаковки теннисных мячей на общую сумму 12 долларов. Всего 6 теннисных мячей. Сколько стоит 1 упаковка теннисных мячей? Сколько стоит 1 теннисный мяч?

26. Переводчик: Итальянский ресторан получил партию из 86 котлет из телятины. Если на блюдо нужно 3 котлеты, сколько котлет останется в ресторане после приготовления как можно большего количества блюд?

Задачи со смешанными операциями со словами

Подходит для: 3-й класс, 4-й класс, 5-й класс

27.Смешивание сложения и вычитания: В библиотеке 235 книг. В понедельник вывозят 123 книги. Во вторник возвращено 56 книг. Сколько сейчас книг?

28. Смешивание, умножение и деление: Есть группа из 10 человек, которые заказывают пиццу. Если каждый человек получает 2 куска, а у каждой пиццы 4 куска, сколько пиццы им следует заказать?

29. Смешивание, умножение, сложение и вычитание: У Ланы 2 пакета по 2 шарика в каждом.У Маркуса 2 сумки по 3 шарика в каждой. Сколько еще шариков у Маркуса?

30. Подразделение смешивания, сложения и вычитания: У Ланы 3 мешка с одинаковым количеством шариков, всего 12 шариков. У Маркуса 3 сумки с таким же количеством шариков, всего 18 шариков. Сколько еще шариков у Маркуса в каждой сумке?

Проблемы с упорядочением и распознаванием чисел

Подходит для: 2-й класс, 3-й класс

31.Подсчет для предварительного умножения: В вашем классе 2 классные доски. Если на каждую классную доску нужно 2 куска мела, сколько всего кусков вам нужно?

32. Подсчет перед предварительным просмотром: В вашем классе 3 классные доски. На каждой доске по 2 мелка. Это означает, что всего есть 6 мелков. Если вы уберете по 1 мелу с каждой доски, сколько всего их будет?

33. Составление чисел: Какое число 6 десятков и 10 единиц?

34.Числа для угадывания: У меня цифра 7 в разряде десятков. У меня четное число вместо единиц. Мне меньше 74. Какой я номер?

35. В поисках заказа: В хоккейном матче Митчелл набрал больше очков, чем Уильям, но меньше очков, чем Остон. Кто набрал больше всего очков? Кто набрал меньше всего очков?

Задачи с дробями

Подходит для: 3-й класс, 4-й класс, 5-й класс, 6-й класс

36.Поиск фракций группы: Джулия пошла в 10 домов на своей улице на Хэллоуин. В 5 домах ей подарили плитку шоколада. В какой части домов на улице Джулии ей дали плитку шоколада?

37. Поиск фракций единицы: Хизер рисует портрет своей лучшей подруги Лизы. Чтобы было легче, она делит портрет на 6 равных частей. Какая дробь представляет каждую часть портрета?

38. Сложение дробей с одинаковыми знаменателями: Ной проходит ⅓ километра до школы каждый день.Он также проходит ⅓ километра, чтобы вернуться домой после школы. Сколько всего километров он проходит?

39. Вычитание дробей с одинаковыми знаменателями: На прошлой неделе Уитни подсчитала количество коробок сока, которые у нее были на школьные обеды. У нее было случая. На этой неделе осталось ⅕ случая. Сколько вина выпила Уитни?

40. Сложение целых чисел и дробей с одинаковыми знаменателями: В обеденное время в кафе-мороженом подавалось 6 ложек шоколадного мороженого, 5 ложек ванили и 2 ложки клубники.Сколько всего шариков мороженого обслужили в салоне?

41. Вычитание целых чисел и дробей с одинаковыми знаменателями: На вечеринке у Хайме было 5 ⅓ бутылок колы, чтобы ее друзья выпили. Она сама выпила бутылки. Ее друзья выпили 3. Сколько бутылок колы осталось у Хайме?

42. Сложение дробей с непохожими знаменателями: Кевин выполнил ½ задания в школе. Вернувшись в тот вечер домой, он выполнил ⅚ другого задания.Сколько заданий выполнил Кевин?

43. Вычитание дробей с непохожими знаменателями: Собирая школьные обеды для своих детей, Пэтти использовала упаковки ветчины. Еще она использовала ½ упаковки индейки. Насколько больше ветчины, чем индейки, использовала Пэтти?

44. Умножение дробей: Во время урока физкультуры в среду ученики пробежали километра. В четверг они пробежали ½ километра, как в среду. Сколько километров пробежали студенты в четверг? Запишите свой ответ дробью.

45. Разделение на фракции: Производитель одежды использует флакона цветного красителя для изготовления одной пары брюк. Производитель вчера использовал бутылки. Сколько пар брюк изготовил производитель?

46. Умножение дробей на целые числа: На этой неделе Марк выпил ⅚ пакета молока. Фрэнк выпил в 7 раз больше молока, чем Марк. Сколько пакетов молока выпил Фрэнк? Запишите свой ответ дробью, целым или смешанным числом.

Десятичные проблемы со словами

Подходит для: 4-й класс, 5-й класс

47.Добавление десятичных знаков: У вас в миске 2,6 грамма йогурта, и вы добавляете еще одну ложку 1,3 грамма. Сколько всего йогурта у вас есть?

48. Вычитание десятичных знаков: У Джеммы было 25,75 грамма глазури для изготовления торта. Она решила использовать только 15,5 грамма глазури. Сколько глазури осталось у Джеммы?

49. Умножение десятичных дробей на целые числа: Маршалл проходит в общей сложности 0,9 км до школы и обратно каждый день. Сколько километров он пройдет через 4 дня?

50.Разделение десятичных дробей на целые числа: Чтобы сделать Пизанскую башню из спагетти, миссис Робинсон купила 2,5 килограмма спагетти. Всего ее ученики смогли построить 10 наклонных башен. Сколько килограммов спагетти нужно для изготовления 1 падающей башни?

51. Смешивание сложения и вычитания десятичных знаков: У Рокко в холодильнике 1,5 литра апельсиновой соды и 2,25 литра виноградной газировки. У Антонио есть 1,15 литра апельсиновой газировки и 0,62 литра виноградной газировки. Насколько больше газировки у Рокко, чем у Анджело?

52.Смешивание умножения и деления десятичных знаков: 4 дня в неделю Лаура занимается боевыми искусствами по 1,5 часа. Учитывая, что в неделе 7 дней, каково ее среднее время занятий в день каждую неделю?

Сравнение и упорядочение словарных задач

Подходит для: Детский сад, 1-й класс, 2-й класс

53. Сравнение 1-значных целых чисел: У вас 3 яблока, а у вашего друга 5 яблок. У кого больше?

54. Сравнение 2-значных целых чисел: У вас 50 конфет, а у вашего друга 75 конфет.У кого больше?

55. Сравнение различных переменных: На детской площадке есть 5 баскетбольных мячей. На детской площадке установлено 7 футбольных мячей. Есть еще баскетбольные мячи или футбольные мячи?

56. Последовательность однозначных целых чисел: У Эрика 0 наклеек. Каждый день он получает еще 1 наклейку. Сколько дней до того, как он получит 3 наклейки?

57. Пропуск по нечетным числам: Натали начала с 5. Она начала счет по пятеркам. Могла ли она сказать число 20?

58.Пропуск по четным числам: Наташа начала с 0. Она считала по восьмеркам. Могла ли она сказать число 36?

59. Последовательность 2-значных чисел: Каждый месяц Джереми добавляет такое же количество карточек в свою коллекцию бейсбольных карточек. В январе у него было 36. В феврале 48. 60 марта. Сколько бейсбольных карточек будет у Джереми в апреле?

Задачи со словом времени

Подходит для: 1-й класс, 2-й класс

66. Преобразование часов в минуты: Джереми помогал своей маме 1 час.Сколько минут он ей помогал?

69. Время добавления: Если вы просыпаетесь в 7:00 утра и вам требуется 1 час 30 минут, чтобы собраться и пойти в школу, в какое время вы придете в школу?

70. Время вычитания: Если поезд отправляется в 14:00. и прибывает в 16:00, сколько времени пассажиры находились в поезде?

71. Определение времени начала и окончания: Ребекка вышла из магазина своего отца, чтобы пойти домой в двадцать семь вечера.Через сорок минут она была дома. Во сколько она приехала домой?

Задачи с деньгами

Подходит для: 1-й, 2-й, 3-й, 4-й, 5-й класс

60. Добавление денег: Томас и Мэтью копят деньги, чтобы вместе купить видеоигру . Томас сэкономил 30 долларов. Мэтью сэкономил 35 долларов. Сколько денег они накопили в общей сложности?

61. Вычитание денег: Томас накопил 80 долларов. На свои деньги он покупает видеоигру.Видеоигра стоит 67 долларов. Сколько денег у него осталось?

62. Умножение денег: Тим получает 5 долларов за доставку бумаги. Сколько у него будет денег после 3-х раздачи бумаги?

63. Разделение денег: Роберт потратил 184,59 доллара на покупку 3 хоккейных клюшек. Если бы каждая хоккейная клюшка имела одинаковую цену, сколько стоила бы 1 клюшка?

64. Сложение денег с десятичными знаками: Вы пошли в магазин и купили жевательную резинку за 1,25 доллара и присоску за 0,50 доллара. Сколько было у вас всего?

65.Вычитание денег с десятичными знаками: Вы пошли в магазин с 5,50 долларами. Вы купили жевательную резинку за 1,25 доллара, плитку шоколада за 1,15 доллара и присоску за 0,50 доллара. Сколько у тебя осталось денег?

67. Применение пропорциональных отношений к деньгам: Якоб хочет пригласить 20 друзей на свой день рождения, что обойдется его родителям в 250 долларов. Если он вместо этого решит пригласить 15 друзей, сколько денег это будет стоить его родителям? Предположим, что отношение прямо пропорционально.

68.Применение процентных соотношений к деньгам: Retta положила 100 долларов США на банковский счет, который приносит 20% годовых. Сколько процентов будет накоплено за 1 год? И если она не снимает деньги, сколько денег будет на счету через 1 год?

Проблемы со словами физических измерений

Подходит для: 1-й, 2-й, 3-й, 4-й класс

72. Сравнение измерений: Линейка Кассандры имеет длину 22 сантиметра. Линейка апреля имеет длину 30 сантиметров.На сколько сантиметров длиннее линейка апреля?

73. Измерения в контексте: Представьте школьный автобус. Какая единица измерения лучше всего описывает длину автобуса? Сантиметры, метры или километры?

74. Добавление измерений: Папа Миши хочет сэкономить на бензине, поэтому он отслеживает, сколько он потребляет. В прошлом году папа Миши использовал 100 литров бензина. В этом году ее отец использовал 90 литров бензина. Сколько всего газа он использовал за два года?

75.Вычитание измерений: Папа Миши хочет сэкономить на бензине, поэтому отслеживает, сколько он потребляет. За последние два года папа Миши использовал 200 литров бензина. В этом году он использовал 100 литров газа. Сколько газа он использовал в прошлом году?

76. Умножение объема и массы: Кира хочет убедиться, что у нее крепкие кости, поэтому она выпивает 2 литра молока каждую неделю. Сколько литров молока выпьет Кира через 3 недели?

77. Разделение объема и массы: Лилиан занимается садоводством, поэтому она купила 1 килограмм земли.Она хочет равномерно распределить почву между двумя растениями. Сколько получит каждое растение?

78. Преобразование массы: Ингер идет в продуктовый магазин и покупает 3 тыквы, каждая из которых весит 500 грамм. Сколько килограммов кабачков купила Ингер?

79. Преобразование объема: У Шэда есть киоск для лимонада, и он продал 20 чашек лимонада. Каждая чашка была 500 миллилитров. Сколько литров всего продала Шад?

80. Длина преобразования: Стейси и Мильда сравнивают свой рост.Рост Стейси 1,5 метра. Милда на 10 сантиметров выше Стейси. Какой рост у Милды в сантиметрах?

81. Расстояние и направление: Автобус отправляется из школы, чтобы отвезти учащихся на экскурсию. Автобус едет 10 километров на юг, 10 километров на запад, еще 5 километров на юг и 15 километров на север. В каком направлении должен ехать автобус, чтобы вернуться в школу? Сколько километров он должен пройти в этом направлении?

Соотношение и процентное соотношение словарных задач

Подходит для: 4-й класс, 5-й класс, 6-й класс

82.В поисках недостающего числа: Соотношение трофеев Дженни и трофеев Мередит составляет 7: 4. У Дженни 28 трофеев. Сколько у Мередит?

83. Поиск недостающих номеров: Соотношение трофеев Дженни и трофеев Мередит составляет 7: 4. Разница между числами — 12. Какие числа?

84. Сравнительные показатели: В младшем школьном оркестре 10 саксофонистов и 20 трубачей. В старшем оркестре школы 18 саксофонистов и 29 трубачей.В каком оркестре соотношение трубачей и саксофонистов больше?

85. Определение процентного соотношения: Мэри опросила учеников своей школы, чтобы определить их любимые виды спорта. 455 из 1200 студентов назвали хоккей своим любимым видом спорта. Какой процент студентов назвал хоккей своим любимым видом спорта?

86. Определение процента изменения: Десять лет назад население Оквилла составляло 67 624 человека. Теперь он на 190% больше. Каково население Оквилля в настоящее время?

87.Определение процентов чисел: На пункте проката коньков 60% из 120 коньков предназначены для мальчиков. Если остальные коньки для девочек, сколько их?

88. Расчет средних значений: В течение 4 недель Уильям вызывался помощником на уроках плавания. Первую неделю он работал волонтером по 8 часов. Он работал волонтером 12 часов на второй неделе и еще 12 часов на третьей неделе. На четвертой неделе он работал волонтером по 9 часов. Сколько часов в среднем он работал волонтером в неделю?

Вероятность и проблемы со связями данных

Подходит для: 4-й класс, 5-й класс, 6-й класс, 7-й класс

89.Понимание предпосылки вероятности: Джон хочет узнать любимое телешоу своего класса, поэтому он опрашивает всех мальчиков. Будет ли выборка репрезентативной или необъективной?

90. Понимание материальной вероятности: Грани на большом количестве кубиков помечены цифрами 1, 2, 3, 4, 5 и 6. Вы бросаете кубик 12 раз. Сколько раз вы должны ожидать, что вам выпадет 1?

91. Изучение дополнительных событий: Цифры от 1 до 50 находятся в шляпе. Если вероятность выпадения четного числа составляет 25/50, какова вероятность НЕ выпадать четное число? Выразите эту вероятность дробью.

92. Исследование экспериментальной вероятности: В магазине пиццы недавно было продано 15 пицц. 5 из этих пицц были пепперони. Отвечая дробью, какова экспериментальная вероятность того, что следующая пицца будет пепперони?

93. Введение в отношения данных: Маурита и Феличе проходят по 4 теста. Вот результаты 4 тестов Мауриты: 4, 4, 4, 4. Вот результаты 3 из 4 тестов Феличе: 3, 3, 3. Если среднее значение Мауриты по 4 тестам на 1 балл выше, чем у Феличе, каков результат? оценка 4-го теста Феличе?

94.Представляем пропорциональные отношения: Store A продает 7 фунтов бананов за 7 долларов. Магазин B продает 3 фунта бананов по цене 6 долларов. В каком магазине выгоднее?

95. Написание уравнений для пропорциональных отношений: Лайонел любит футбол, но у него проблемы с мотивацией к тренировкам. Итак, он стимулирует себя с помощью видеоигр. Существует пропорциональная зависимость между количеством упражнений, которые Лайонел выполняет, в x , и тем, сколько часов он играет в видеоигры, в x .Когда Лайонел выполняет 10 упражнений, он 30 минут играет в видеоигры. Напишите уравнение отношения между x и y .

Геометрические задачи со словом

Подходит для: 4-й класс, 5-й, 6-й, 7-й, 8-й классы

96. Представляем Периметр: В театре 4 стула в ряд. Всего 5 рядов. Если использовать строки в качестве единицы измерения, каков периметр?

97. Зона представления: В театре 4 стула в ряд.Всего 5 рядов. Сколько всего стульев?

98. Введение Том: Аарон хочет знать, сколько конфет может вместить его контейнер. Контейнер имеет высоту 20 сантиметров, длину 10 сантиметров и ширину 10 сантиметров. Каков объем контейнера?

99. Понимание 2D-форм: Кевин рисует фигуру с 4 равными сторонами. Какую форму он нарисовал?

100. Обнаружение периметра 2D-форм: Митчелл написал свои домашние задания на листе квадратной бумаги.Каждая сторона бумаги по 8 сантиметров. Какой периметр?

101. Определение площади 2D-форм: Одна торговая карточка имеет длину 9 см и ширину 6 см. Какая у него площадь?

102. Что такое 3D-фигуры: Марта рисует фигуру с 6 квадратными гранями. Какую форму она нарисовала?

103. Определение площади поверхности трехмерных фигур: Какова площадь поверхности куба шириной 2 см, высотой 2 см и длиной 2 см?

104.Определение объема 3D-форм: Контейнер для конфет Аарона имеет высоту 20 сантиметров, длину 10 сантиметров и ширину 10 сантиметров. Контейнер Брюса имеет высоту 25 сантиметров, длину 9 сантиметров и ширину 9 сантиметров. Найдите объем каждого контейнера. В зависимости от объема, чей контейнер может вместить больше конфет?

105. Определение прямоугольных треугольников: Треугольник имеет следующие длины сторон: 3 см, 4 см и 5 см. Этот треугольник прямоугольный?

106.Определение равносторонних треугольников: Треугольник имеет следующие длины сторон: 4 см, 4 см и 4 см. Что это за треугольник?

107. Определение равнобедренных треугольников: Треугольник имеет следующие длины сторон: 4 см, 5 см и 5 см. Что это за треугольник?

108. Определение треугольников из чешуи: Треугольник имеет следующие длины сторон: 4 см, 5 см и 6 см. Что это за треугольник?

109. Определение периметра треугольников: Луиджи построил палатку в форме равностороннего треугольника.Периметр 21 метр. Какова длина каждой стороны палатки?

110. Определение площади треугольников: Какова площадь треугольника с основанием в 2 единицы и высотой 3 единицы?

111. Применение теоремы Пифагора: Прямоугольный треугольник имеет длину одной стороны без гипотенузы 3 дюйма и длину гипотенузы 5 дюймов. Какова длина другой стороны без гипотенузы?

112. Определение диаметра круга: Жасмин купила новый круглый рюкзак.Его площадь составляет 370 квадратных сантиметров. Какой диаметр у круглого рюкзака?

113. Поиск области круга: Круглый щит Капитана Америки имеет диаметр 76,2 сантиметра. Какова площадь его щита?

114. Поиск радиуса круга: Скайлар живет на ферме, где его отец держит круглый кукурузный лабиринт. Кукурузный лабиринт имеет диаметр 2 километра. Каков радиус лабиринта?

Переменные задачи со словами

Подходит для: 6 класс, 7 класс, 8 класс

115.Определение независимых и зависимых переменных: Виктория печет кексы для своего класса. Количество кексов, которые она готовит, зависит от того, сколько у нее одноклассников. Для этого уравнения м — это количество кексов, а c — количество одноклассников. Какая переменная является независимой, а какая зависимой?

116. Написание переменных для сложения: В прошлом футбольном сезоне Триш забила g голов.Алекса забила на 4 гола больше, чем Триш. Напишите выражение, показывающее, сколько голов забила Алекса.

117. Написание выражений переменных для вычитания: Элизабет ест здоровый, сбалансированный завтрак b раз в неделю. Мэдисон иногда пропускает завтрак. В целом Мэдисон съедает на 3 завтрака меньше в неделю, чем Элизабет. Напишите выражение, показывающее, сколько раз в неделю Мэдисон завтракает.

118. Написание выражений переменных для умножения: В прошлом хоккейном сезоне Джек забил g голов.Патрик забил вдвое больше голов, чем Джек. Напишите выражение, показывающее, сколько голов забил Патрик.

119. Написание выражений переменных для подразделения: У Аманды c плиток шоколада. Она хочет равномерно распределить плитки шоколада между 3 друзьями. Напишите выражение, показывающее, сколько плиток шоколада получит один из ее друзей.

120. Решение уравнений с двумя переменными: Это уравнение показывает, как сумма, которую Лукас зарабатывает на внешкольной работе, зависит от того, сколько часов он работает: e = 12h .Переменная h представляет, сколько часов он работает. Переменная e представляет, сколько денег он зарабатывает. Сколько денег заработает Лукас, проработав 6 часов?

Как легко составлять свои собственные математические задачи со словом и рабочие листы с задачами со словом

Вооружившись 120 примерами, чтобы зажечь идеи, создание собственных задач по математике со словом может заинтересовать ваших учеников и обеспечить согласованность с уроками. Do:

  • Ссылка на интересы студентов: Обрамляя свои текстовые проблемы интересами студентов, вы, вероятно, привлечете внимание.Например, если большая часть вашего класса любит американский футбол, задача измерения может включать расстояние броска известного квотербека.
  • Сделайте вопросы актуальными: Написание словесной задачи, отражающей текущие события или проблемы, может заинтересовать учащихся, предоставив им четкий, осязаемый способ применения своих знаний.
  • Включите имена учащихся: Назовите символы вопроса в честь учащихся — это простой способ сделать предмет понятным, помогая им справиться с проблемой.
  • Будьте явными: Повторение ключевых слов определяет вопрос, помогая учащимся сосредоточиться на основной проблеме.

Не нужно:

  • Тест на понимание прочитанного: Цветочный выбор слов и длинные предложения могут скрыть ключевые элементы вопроса. Вместо этого используйте краткие фразы и лексику на уровне своего класса.
  • Сосредоточьтесь на схожих интересах: Слишком много вопросов, связанных с интересами, такими как футбол или баскетбол, может оттолкнуть или оттолкнуть некоторых студентов.
  • Особые опасения: Включение ненужной информации вводит еще один элемент решения проблем, подавляющий многих учеников начальной школы.

Ключ к дифференцированному обучению, словесные задачи, которые студенты могут связать и контекстуализировать, вызовут больший интерес, чем общие и абстрактные.

Заключительные мысли о математических задачах со словами

Вероятно, вы получите максимальную отдачу от этого ресурса, если будет использовать задачи в качестве шаблонов, слегка изменяя их, применяя приведенные выше советы. Таким образом, они будут более актуальны и интересны для ваших учеников.

Тем не менее, наличие 120 задач по математике, соответствующих учебной программе, на кончиках ваших пальцев, должно помочь вам решать задачи по развитию навыков и давать задания, заставляющие задуматься.

Результат?

Более глубокое понимание того, как ваши ученики обрабатывают контент, и демонстрация понимания, что дает информацию о вашем текущем подходе к обучению.

30 вопросов, которые вам понадобятся, чтобы сдать 6-й класс по математике

Если ваша карьера не привела вас к математике или инженерии, скорее всего, вы не занимаетесь тригонометрическими функциями или сложным исчислением на регулярной основе.Но вы, вероятно, используете основные математические концепции, которые вам приходилось осваивать в начальной школе почти каждый день, независимо от того, пытаетесь ли вы расставить мебель в гостиной или как сократить время в пути.

Тем не менее, хотя вы используете множество понятий из шестого класса, вероятно, прошло некоторое время с тех пор, как вы пересмотрели то, что на самом деле знаете, — и вы можете быть удивлены своими математическими слепыми пятнами. Чтобы проверить то, что вы знаете — и, что более важно, чего вы не знаете, — вот 30 вопросов, на которые любой ученик с отличной оценкой сможет ответить.(Примечание: они становятся все сложнее!) А чтобы узнать больше о тестах, которые вы можете или не сможете сдать, посмотрите, как наш репортер сдал президентский когнитивный тест (и вот они набрали).

Знак «-» здесь означает, что это отрицательные числа, что означает, что они меньше нуля.

Отрицательный 2 больше отрицательного 5. Чтобы помочь вам думать об отрицательных числах, представьте себе настольную игру, в которой «больше» означает «ближе к финишу».»Представьте, что красная и синяя фигуры начинаются с одной и той же точки (ноль). Красная фигура должна переместиться назад на 2 деления (-2). Синяя фигура должна переместиться на 5 делений назад (-5). Какая из них ближе всего к финишу? Красный!

Готов поспорить, вы не знали, что вам придется учить новый словарный запас в классе математики! А чтобы пополнить свой нематематический лексикон, начните с запоминания этих 47 крутых иностранных слов, которые заставят вас казаться безумно сложными.

Дроби — это числа, разделенные по вертикали знаком — или /.В десятичных числах есть точка, которая в математике называется десятичной точкой. Целые числа — это числа без дробей и десятичных знаков, даже если они отрицательные. И чтобы узнать о других способах отточить свой ум, попробуйте эти головоломки, чтобы узнать, умнее ли вы астронавта.

Подсказка: если вы не помните, как переводить проценты в десятичные числа, просто удалите знак процента и сдвиньте десятичную запятую на два разряда влево.

0.85 x 21 = 17,85 Или, выражаясь в предложении, 85% от (x) 21 равно (равно) 17,85.

Разберитесь с десятичными числами — вам нужно знать, как их складывать, вычитать, умножать и делить.

Сложение и вычитание десятичных чисел не так сложно, как кажется. Просто не забудьте выровнять десятичные точки и поставить нули в любые правые столбцы, в которых нет чисел, если это необходимо (например: 8,563 0 + 4,8292). А чтобы узнать о других головоломках, посмотрите, как наш корреспондент сдал когнитивный экзамен НФЛ — и вот что они узнали.

Это простое алгебраическое выражение. Алгебра — это просто использование символов в математике.

Любая буква может использоваться как переменная (или неизвестное число). В данном случае это строчная «u». Так что просто введите число для переменной, и вы получите 47 — 23 = 24. Затем, укрепите свой мозг с помощью этой научно доказанной уловки!

Это называется алгебраическим уравнением , потому что числа по обе стороны от знака равенства должны быть одинаковыми.

Поскольку мы знаем, что w обозначает неизвестное число и числа по обе стороны от знака равенства должны совпадать, мы спрашиваем: «Какое число можно вычесть из 9, чтобы получить 8?»

Эти математические утверждения называются неравенствами. В отличие от уравнений, числа по обе стороны от <или> не обязательно должны быть равными.

Если вы прочитаете это вслух, вы скажете: «6 меньше 7», поэтому <- это символ «меньше».Если вы перевернете его и напишете «7> 6», он скажет «7 больше 6». Итак,> — это символ «больше».

Независимые и зависимые переменные также используются в научных экспериментах.

Независимая переменная — это то, что вы контролируете — в этом случае вы можете решить, сколько дел по дому нужно сделать. Зависимая переменная — это то, что зависит от независимой переменной — в этом случае, сколько денег вы зарабатываете, зависит от того, сколько работы вы выполняете.Это число будет изменяться при изменении независимой переменной. А чтобы узнать о других способах повышения резкости, начните с передовых видеоигр, которые сделают вас умнее.

Изучение треугольников называется тригонометрией. В шестом классе вы не выучите всего, что нужно знать о треугольниках, но вы начнете хорошо!

Площадь треугольника равна его основанию, умноженному на его высоту (5 x 4 = 20), разделенной пополам (20 ÷ 2 = 10). Вы увидите, что эта формула записывается как A = ½ bh .

Графики — отличный способ визуализировать и сравнивать числа. Гистограммы (или диаграммы) — один из наиболее распространенных типов.

Сначала мы смотрим на ось Y (вертикальную), чтобы найти то, что нам сказали искать: круговые диаграммы. Затем мы перемещаемся по оси X (горизонтальной), чтобы увидеть, где заканчивается полоса: 2. Это означает, что было два случая, когда люди говорили «мой любимый график — круговая диаграмма».

Среднее значение группы чисел иногда называют «средним».«

Среднее значение рассчитывается путем сложения чисел в наборе (5 + 3 + 6 + 2 = 16) и деления полученного ответа на количество элементов в наборе (16 ÷ 4 = 4).

Чтобы разделить дроби, вы умножаете обратную. Не волнуйтесь, это не так сложно, как кажется!

Умножьте числитель или верхнее число первой дроби (4) на знаменатель или нижнее число второй дроби (2), чтобы получить числитель ответа (4 x 2 = 8).Затем умножьте знаменатель первой дроби (5) на числитель второй дроби (1), чтобы получить знаменатель ответа (5 x 1 = 5). Затем упростите 8/5 до 1⅗.

Абсолютные значения обычно записываются между ||, поэтому вы также можете написать этот вопрос как | -8 | знак равно

Абсолютное значение любого числа, положительного или отрицательного, всегда положительно. Вы можете думать об этом как о расстоянии от нуля на числовой прямой. И 8, и -8 имеют одинаковое абсолютное значение 8, поскольку они оба на 8 целых чисел от нуля.А чтобы получить больше способов отточить свой ум, начните с «50 лучших продуктов для мозга».

Этот процесс называется объединением подобных терминов.

Пока числа объединены с одной и той же переменной (в данном случае a ), вы можете складывать, вычитать, умножать или делить их, как если бы переменной там нет. Вы даже можете думать о переменной как о физическом объекте. Если у вас есть 4 яблока, и я дам вам еще 2 яблока, сколько яблок у вас сейчас?

Параллелограмм — это четырехсторонняя фигура, состоящая из двух пар параллельных линий.

Вы можете найти площадь параллелограмма, умножив его основание на высоту. Эта формула также может быть записана как A = bh , поэтому 5 x 4 = 20.

В математике верхний индекс — это показатель степени, что означает, что для поиска ответа потребуется повторное умножение.

Показатель показывает, сколько раз нужно умножить базовое число само на себя, чтобы получить полное число. Его обычно используют для упрощения длинных чисел, так как, например, 10⁹ намного проще работать, чем 1 000 000 000 (девять нулей).

Вам нужно будет знать, как найти множители любого заданного числа, а затем сравнить их.

Множители 36 — это 1, 2, 3, 4, 6, 9, 12, 18 и 36. Множители 12 — это 1, 2, 3, 4, 6 и 12. Наибольшее общее число. составляет 12.

Ящичковая диаграмма — это график, позволяющий показать разброс и форму данных. «Данные» — это просто причудливое слово для набора чисел, часто чисел, которые соответствуют результатам опроса или эксперимента.

Прямоугольник на прямоугольной диаграмме показывает средние 50% диапазона чисел. Внутри этого поля находится линия, определяющая медиану или значение, которое было бы прямо посередине, если бы вы выровняли все числа от наименьшего к наибольшему. Например, средние 50% диапазона 2, 3, 6 и 8 будут 3–6, а медиана — 4,5. На приведенном выше прямоугольном графике линия для медианы отображается между 8 и 10 примерно на уровне 8,5.

Умножение и деление десятичных чисел не так сложно, если вы удалите десятичную дробь… но не забудьте вернуть ее обратно!

Сначала представьте, что 1.92 — это целое число: 192 с двумя десятичными знаками. Затем разделите 192 на 3, чтобы получить 64. Теперь вставьте два десятичных знака обратно, чтобы получить окончательный ответ 0,64.

Знайте свои десятки, сотни и тысячи, но не путайте их с десятыми, сотыми и тысячными долями!

2 находится в столбце сотен, а следующее число справа (столбец десятков) — 9, что означает, что вам нужно округлить в большую сторону.

Этот тип графика называется гистограммой, но вы можете использовать те же термины для описания точечных диаграмм, гистограмм и ящичных диаграмм.

Поскольку форма, которую создают эти данные, является (приблизительно) одной и той же формой по обе стороны от центральной оси (в данном случае нулевая точка на оси X), она симметрична. Поскольку он достигает наивысшего пика на оси Y посередине, мы говорим, что он имеет форму колокола.

У вас останется несколько дюймов.

В одном футе 12 дюймов, поэтому 272 делить на 12 дает 22 с остатком 8.

Множители и множители тесно связаны.

Наименьшее общее кратное (иногда сокращенно НОК) — это наименьшее число, которое делится на два (или более) рассматриваемых целых числа. Таким образом, 208 — это наименьшее число, которое можно разделить поровну как на 16 (208 ÷ 16 = 13), так и на 26 (208 ÷ 26 = 8).

Площадь поверхности — это сумма площадей всех фигур, составляющих внешнюю часть трехмерной фигуры.

Прямоугольные формы, как и картонные коробки, имеют 6 внешних граней: верхнюю и нижнюю, переднюю и заднюю, левую и правую.Формула для расчета этой площади: (длина x ширина) x 2 + (длина x высота) x 2 + (высота x ширина) x 2. Здесь это означает (4 x 3) x 2 = 24, (4 x 7) x 2 = 56 и (7 x 3) x 2 = 42. Сложите их вместе: 24 + 56 + 42 = 122 кв.

Чтобы складывать или вычитать дроби, они должны иметь одинаковый знаменатель.

Сначала вы должны найти наименьший общий знаменатель или наименьшее число, которое делится на оба нижних числа дробей.Для 12 и 5 это число равно 60. Затем вы должны преобразовать каждый номинатор. Так как вам нужно умножить 12 на 5, чтобы получить 60, умножьте 7 x 5, чтобы получить 35/60. Поскольку вам нужно умножить 5 на 12, чтобы получить 60, умножьте 2 на 12, чтобы получить 24/60. Теперь их можно сложить: 35/60 + 24/60 = 59/60. Вы не можете дальше упрощать 59/60, так что это ваш ответ!

Важно знать, как наносить точки на координатную плоскость. Вас также могут попросить найти область фигур на координатной сетке.

Первое число пары — это положение на оси X (горизонтальной); второе число — это положение на оси Y (вертикальной). Ваша сетка должна выглядеть так, как показано выше.

Объем формы — это количество материала, которое может поместиться внутри нее. Двумерные формы, такие как квадраты, не имеют объема, но трехмерные формы, такие как кубы, обладают.

Объем любой прямоугольной коробки можно найти, умножив длину, , ширину, , высоту, .Поскольку все ребра куба имеют одинаковую длину, 10 x 10 x 10 = 1000.

Пи — постоянное число, а не вкусный десерт! Знайте, как использовать число пи (π) для вычисления площади или длины окружности.

Найдите длину окружности, умножив число пи (π = 3,14) на диаметр (7,7 фута).

Эта задача требует, чтобы вы преобразовали возрасты в смешанные числа и вычли их.

Чтобы вычесть смешанные числа, преобразуйте их в неправильные дроби и выполните ту же процедуру, что и для сложения и вычитания дробей.Поскольку три месяца — это года, а шесть месяцев — ½ года, проблема решается до 9¼ — 2 ½ года. Преобразуйте оба в четверти, так что 9 = 37/4 и 2 ½ = 10/4. 37/4 — 10/4 = 27/4. Упростите 27/4 до 6¾. Три квартала в году составляют девять месяцев, поэтому ответ — 6 лет и 9 месяцев.

Так же, как укус превращает человека в зомби, вы превращаете эти числа в дроби!

Соотношение говорит нам, что на каждые 2 зомби приходится 3 человека, что дает нам группу из 5 (2 + 3) человек.Разделите общую сумму (85) на группы по 5 человек, получив 17 групп. Умножьте их на исходное соотношение, чтобы получилось (17 x 2) 34 зомби и (17 x 3) 51 человек.

Чтобы узнать больше удивительных секретов о том, как жить своей лучшей жизнью, нажмите здесь , чтобы подписаться на нашу БЕСПЛАТНУЮ ежедневную рассылку новостей!

.

Добавить комментарий

Ваш адрес email не будет опубликован. Обязательные поля помечены *